*NURSING > EXAM > Med-Surg Certification Practice 271 Questions with Verified Answers,100% CORRECT (All)

Med-Surg Certification Practice 271 Questions with Verified Answers,100% CORRECT

Document Content and Description Below

Med-Surg Certification Practice 271 Questions with Verified Answers A newly diabetic patient will receive instructions from his nurse. Which of the following actions should the nurse take first? ... A. Provide the patient with brochures for making smart choices for cooking/baking. B. Ask the patient what they know about diabetes. C. Instruct the patient on exercise or activities that will increase their metabolism. D. Teach them how to use the glucometer. - CORRECT ANSWER B. First ask the patient what they know about diabetes. *ACTION questions ask for intervention, NOT assessment The post hysterectomy patient with calls the clinic stating that she has been having difficulty with incontinence. Which of the following statements if made by the patient, indicates that further teaching is needed? A. I drink 8 oz. of fluid on the even hours while awake. B. I will attempt to void 30 minutes after drinking 200 cc. of fluid C. I will only drink fluid when I eat my meals D. I will set the timer to remind me when to void - CORRECT ANSWER C. A patient saying that he or she will only drink fluids when eating meals indicates that they don't know what to do in case of incontinence. FURTHER TEACHING indicates that the patient needs more information An RN assesses a patient in the Cancer Center with a diagnosis of terminal cancer. The patient states to the RN that he is short of breath, has no appetite and hurts everywhere. Which of the following statements by the nurse is best? A. Tell me how your family is coping B. Show me where it does not hurt C. I'll teach you relaxation techniques D. Your physician should increase your pain medication - CORRECT ANSWER C A clinical nurse returns to the desk to find 4 phone messages. Which of the following messages should the nurse respond to first? A. A post cervical laminectomy patient complaining of sudden difficulty talking. B. A patient with multiple sclerosis complaining of change in peripheral vision. C. A patient with a herniated disc complaining of consistent back pain. D. A patient with a cast due to a fracture of the right tibial bone complaining of tingling toes. - CORRECT ANSWER A B and C are expected symptoms of the patient's condition. A and D are unexpected. D is less critical than A. Best answer is A. A 75 year old patient complains to the clinical nurse that he is having difficulty sleeping. Which of the following actions by the nurse is most appropriate? A. Determine the patient's usual sleeping and waking patterns. B. Suggest that the client abstain from alcohol and caffeine before bed time. C. Recommend that client establish a bedtime routine. D. Ask how much sleep the client required before retirement. - CORRECT ANSWER A B and C are recommendations that the RN can share after he/she determines the patient's sleeping and waking patterns. So remove B and C. D is an inquiry, but is concerned about the past (when the person retired). Remove D. A is the best answer. The nurse cares for a patient several hours after a thyroidectomy. The RN observes that the patient is diaphoretic and confused. The vital signs are: Temp = 102° (38.9 ° C); BP is 160/90; pulse is 110, respiration is 22. The nurse expects the physician to order which of the following? A. NPO, Dextran 10 ml/kg. IV, 02 4 L/min. B. Propranolol (Inderal) 1 mg. IV, D5W 125 cc/hr IV, propylthiaouracil (PTU) 200 mg po qld. C. Levothyroxine Sodium (Synthroid) 100 mcg IV, Lactated Ringer's 125 cc/hr IV, and ciprofloxacin (cipro) 500 mg. po bid. D. Morphine sulfate 2.5 mg. IV q. 4 hrs. prn, 0.9% NACl. - CORRECT ANSWER B A 63 year old female patient contacts the clinic to report that she is has been extremely fatigued, is sleeping 12 hours a night, is always cold and her hair is falling out. Which of the following responses by the nurse is most appropriate? A. You will have more energy if you walk 30 min every day. B. You don't need to worry. Sleeping a lot is good for you. C. Come in and see your health care provider today. D. You should eat more green leafy vegetables. - CORRECT ANSWER C Need more info The RN performs blood pressure screening at the local community center. Which of the following individuals is most likely to suffer from a cerebrovascular accident? A. A 30 year old man who is an account executive B. A 40 year old man who jogs four times a week. C. A 60 year old man whose father had a stroke. D. A 75 year old man who takes multi-vitamins. - CORRECT ANSWER C For the patient with a new tracheostomy, the nurse must be alert to which early complication? a) Decannulation b) Infection c) Bleeding d) Tracheomalacia - CORRECT ANSWER C Common acute risks of tracheostomy include bleeding, airway loss, damage to adjacent structures, and failure of the chosen technique to achieve successful airway EVERYTHING 24 HR POST SURGERY, FIRST COMPLICATION IS BLEEDING. Infection comes 2-3 days later. When a pulse oximetry monitor indicates that a patient has a drop in SpO2 from 96% to 85% over 4 hours, the nurse will first: 1. Request an order for stat ABGs 2. Start the patient on oxygen by NC at 2L/minute 3. Notify the physician of the change 4. Check the position of the probe on patient's finger or earlobe - CORRECT ANSWER 4 Pulse oximetry is inaccurate if the probe is loose, or in the presence of poor circulation, nail polish, or dark skin pigmentation. To decrease a patient's shortness of breath and a sense of impending doom during an asthma attack, the nurse will: A. Place the patient on a cardiac monitor and observe from the nurses' station. B. Let the patient rest alone in a quiet, calm environment. C. Reassure the patient that the doctor will arrive soon D. Stay with the patient and encourage pursed-lip breathing. - CORRECT ANSWER D The patient experiencing an acute asthma exacerbation will be fearful and anxious. It is important for the nurse to stay with the patient for ongoing assessment and to provide a calm environment. Helping the patient breathe with pursed lips will facilitate the expiration of trapped air and help the patient regain control of his or her breathing. A 68 year old woman is admitted to the surgical unit after a gastric resection for gastric cancer. She states that she doesn't want anyone to see her in this condition, even her husband. This nurse's best response is: A."Patients' rights protect your privacy. I won't allow anyone in." B."I've seen gastric cancer patients who look much worse. Don't worry." C."Would you like to talk to a social worker?" D."What about your condition worries you?" - CORRECT ANSWER D The nurse should allow the patient to share her concerns and express her feelings in a nonjudgmental environment. If there is psych/behavioral component- AFFIRM AND VALIDATE, NO PROBING A patient with a tricuspid valve disorder will have impaired blood flow between the: A. Vena cava and right atrium B. Left atrium and left ventricle C. Right atrium and right ventricle D. Right ventricle and pulmonary artery - CORRECT ANSWER C Tricuspid valve is located between right atrium and right ventricle MITRAL VALVE- left atrium and left ventricle If the Purkinje system is damaged, conduction of the electrical impulse is impaired through the: A. Atria B. AV node C. Ventricles D. Bundle of His - CORRECT ANSWER C Purkinje fibers provide a high-speed distribution of excitation throughout the ventricular myocardium; if the pattern of contraction is disrupted, the ventricles will not efficiently eject blood Order of impulse is SA node to AV node to Bundle of His to bundle branch to purkinje fibers The P-wave on an EKG is representative of: A. Ventricular depolarization B. Atrial depolarization C. Atrial repolarization D. Ventricular repolarization - CORRECT ANSWER B. Atrial depolarization Ventricular repolarization is represented by the QRS There is no distinctly visible wave representing atrial repolarization in the ECG because it occurs during ventricular depolarization, ventricular repolarization is shown as the interval between the start of the QRS complex and the end of the T wave A patient admitted with an abscess in her left thigh undergoes incision and drainage. 3 days later, she develops fever (temperature of 103°F), chills, lethargy, and shortness of breath. Infectious endocarditis is suspected. In addition to blood cultures, the nurse would expect which of these tests? A. Transesophageal echocardiogram B. Computed tomography of the chest C. Cardiac stress test D. Cardiac enzymes - CORRECT ANSWER A. Echocardiogram to visualize cardiac structures. Which of the following is a sign of poor perfusion? A. Mean arterial pressure <80 mmHg B. Urine output <0.5 ml/kg/hr for 2 consecutive hours C. Lactate levels > 1mmol/L D. Systolic BP of 150 mmHg - CORRECT ANSWER B UO should be greater or equal to .5ml per kg per hour. MAPs should be between 65-85 Lactate levels should be less than 2.3 SBP of 150 is high but does not indicate poor perfusion When performing discharge teaching with a patient who is taking warfarin sodium (Coumadin), the nurse learns that patient used complementary therapies at home. Which of the following substances is most concerning? A. Echinacea B. Vitamin B12 C. Vitamin D D. Garlic - CORRECT ANSWER D. Garlic When taking warfarin, caution with - garlic, ginseng, ginkgo, ibuprofen, pain drugs, vitamin E Garlic may increase bleeding, so supplemental garlic may cause a synergic effect when combined with coumadin For the same patient - who is being discharged and is taking warfarin (Coumadin), the nurse should also emphasize that: A. Regular follow-up with healthcare provider is essential B. Aspirin would be recommended for headache therapy C. The medication can be taken any time of the day D. Diet should be high in iron content - CORRECT ANSWER A Discuss INR. High INR is a risk for bleeding (GI, hemorrhagic stroke, etc) Aspirin also being a blood thinner can be too much blood thinning action Timing of dose needs to be consistent Iron, mag, and zinc bind with coumadin and decrease absorption, it is not an absolute contraindication, but it recommended that these supplements be taken two hours apart from coumadin Which of the following patient teaching points should the nurse include when providing discharge instructions to a patient with a new permanent pacemaker and the caregiver (select all that apply)? A. Avoid or limit air travel B. Take and record a daily pulse rate C. Obtain and wear a Medic Alert ID or bracelet at all times D. Avoid lifting arm on the side of the pacemaker above shoulder E. Avoid microwave ovens because they interfere with pacemaker function - CORRECT ANSWER B, C, D Back in the day, persons with pacers needed to carry a medical device ID card as proof a pacer when they would go through security, as security heightened and body scans became normal this isn't really needed anymore Daily record keeping of HR is recommended as is wearing a med alert id bracelet, patients should NOT lift above the shoulder for a period of time so that the leads can stabilize in the body without being disrupted (about 2-4 weeks) Microwaves and most other household appliances pose no risk but extreme magnetic forces do Causes of secondary hypertension would include: A. Alcohol abuse B. Diabetes C. Obesity D. Renal artery stenosis - CORRECT ANSWER D Renal stenosis → narrowing of arteries that carry blood to the kidneys → less blood to the kidneys → raises blood pressure A 60-year-old African American male who works as a soccer coach, is admitted with hypertension. On admission his blood pressure was 210/108 mmHg. His last total cholesterol level was 156 mg/dl, and a random blood glucose was 110 mg/dl. Which is unalterable risk factor for hypertension? A. His blood glucose level B. His cholesterol level C. His occupation D. His ethnicity - CORRECT ANSWER D Other risk factors: elevated serum lipids, DM, sedentary lifestyle A patient who has type 2 diabetes is prescribed carvedilol (Coreg) for hypertension. The nurse monitors for A. Hypoglycemia B. Hyperglycemia C. Hypocalcemia D. Hypercalcemia - CORRECT ANSWER A. Hypoglycemia In patients with DM, Coreg may increase the effects of hypoglycemic drugs (prevent adrenalin from stimulating the liver to make glucose) and mask symptoms of hypoglycemia. Which of the following is an AHA recommendation for patients with coronary artery disease for stroke prevention? A. Fibrate therapy B. Heparin therapy C. Beta-blocker therapy D. Aspirin therapy - CORRECT ANSWER D. Aspirin therapy This is a core measure for patients with CAD Heparin is not indicated nor is it really sustainable for home use, betablockers don't decrease your risk of stroke directly although it does reduce bp which reduces risk of stroke A client complains of crushing chest pain that radiates to his left arm. Which of the following treatments should you anticipate: A. Aspirin, oxygen, nitroglycerin, and morphine B. Aspirin, oxygen, nitroglycerin, and codeine C. Oxygen, nitroglycerin, meperidine, and thrombolytics D. Aspirin, oxygen, nitroprusside, and morphine - CORRECT ANSWER A. Aspirin, oxygen, nitroglycerin, and morphine MONA A patient is recovering from an uncomplicated MI. Which of the following rehabilitation guidelines is a priority to include in the teaching plan? A. Refrain from sexual activity for a minimum of 3 weeks. B. Plan a diet program that aims for a 1- to 2-pound weight loss per week. C. Begin an exercise program that aims for at least five 30-minute sessions per week. D. Consider the use of erectile agents and prophylactic NTG before engaging in sexual activity. - CORRECT ANSWER C Avoid intercourse for 4-6 weeks The need for a weight loss plan would be patient specific You do not want to mix nitrates and ED meds because this can cause and unsafe drop in BP The most common finding in individuals at risk for sudden cardiac death is: A. aortic valve disease B. mitral valve disease C. left ventricular dysfunction D. atherosclerotic heart disease - CORRECT ANSWER C. left ventricular dysfunction Those with LV dysfunction are at high risk for ventricular arrythmias. Typically, the LV dysfunction is secondary to MI, cardiomyopathy or advanced heart failure. A patient with a history of myocardial infarction (MI) complains of chest pain while walking to the restroom. Which action should the nurse perform first? A. Assess patient's vital signs B. Instruct patient to sit down C. Obtain a STAT EKG D. Call the physician - CORRECT ANSWER B. Instruct patient to sit down Having the patient sit down reduces myocardial oxygen demand to help relieve pain and take stress off the heart. The other actions are important, but would come after having the patient sit down to ensure safety. The key diagnostic test for heart failure is: A. Serum potassium B. B-type natriuretic peptide C. Troponin I D. Cardiac enzymes - CORRECT ANSWER B. B-type natriuretic peptide Hyperkalemia can be common in heart failure secondary to treatments like ACE inhibitors and betablockers can cause the kidneys to retain potassium, similarly diuretic can be used to treat fluid overload associated with heart failure, the desired effect achieving of high urine output to decrease preload can also lead to low potassium levels Troponin is the hallmark diagnostic test for acute coronary syndrome, has no value with HF, same goes for cardiac enzymes A nurse is instructing a patient taking diuretics for heart failure about foods that are high in potassium to include in her daily diet. The nurse concludes that additional education is needed when the client states that the food highest in potassium is: A. Spinach B. Apples C. Avocado D. Sweet potatoes - CORRECT ANSWER B. Apples From this list avocadoes have the most, followed by sweet potatoes, then spinach. Apples have the least Mr. Jones is 72 years old and admitted with an exacerbation of left-sided heart failure. The nurse would expect to see which of these signs or symptoms? A.Hepatomegaly B. Enlarged spleen C. Oliguria D. Ascites - CORRECT ANSWER C. Oliguria Left-sided HF - sodium and water retention and peripheral edema. Decreased blood flow to the kidneys can lead to oliguria. Ascites, hepatomegaly, enlarged spleen are signs of right-sided heart failure. LEFT- Lungs and Kidneys Mr. Jones develops acute pulmonary edema. In anticipation of the arrival of the Rapid Response Team, the nurse should take which of these measures? (select all that apply) A. Place Mr. Jones in high Fowler's position. B. Apply high-flow oxygen. C. Administer nitroglycerin sublingually. D. Insert an indwelling urinary catheter. - CORRECT ANSWER A, B This patient is going to feel like they're drowning in their lungs, and they are. Set the patient upright immediately and supplement O2. Nitroglycerine is a vasodilator and could be used to decrease afterload and strict urine output measuring will be ideal when diuretic treatment is initiated but these are not your initial steps at this time. In preparing for discharge, Mr. Jones should receive education about which of these types of medication? A .Beta-adrenergic agonists B. Angiotensin-converting enzyme (ACE) inhibitors C. Nonsteroidal anti-inflammatories D. Prokinetic agents - CORRECT ANSWER B. ACE inhibitors Ace inhibitors are commonly used to treat HF as are ARBs, beta blockers, calcium channel blockers, and diuretics Beta-adrenergic agonists are used to help with dilating airways but breathing difficulty in HF is a secondary issue to fluid build up in the heart and lungs NOT inflammation or obstruction of the airway NSAIDS and prokinetic agents like Reglan offer no value here A patient with heart failure has been started on metoprolol 12.5mg daily. The nurse should report serious adverse reactions to the medication including: A. Crackles in lungs B. Edema C. Dry, hacking cough D. Dry mouth - CORRECT ANSWER A. Crackles in lungs Indication of exacerbation of HF symptoms → possible complication of starting beta blockers due to the decrease in inotropic response caused by the medication. Edema is also a sign of worsening HF, but is not as specific as lung crackles. Betablockers do cause cough and dry mouth but these are not serious adverse reactions; They're just annoying and anticipated. A patient with HF and atrial fibrillation is treated with a digitalis glycoside and a loop diuretic. To prevent possible complications of this combination of drugs, the nurse needs to: A. monitor serum potassium levels. B. keep an accurate measure of intake and output. C. teach the patient about dietary restriction of potassium. D. withhold digitalis and notify health care provider if heart rate is irregular. - CORRECT ANSWER A. monitor serum potassium levels. Loop diuretics and most diuretics will deplete potassium as they effectively reduce preload by increase urine output. Yes, you do want to measure I&Os, but the question asks about complications not verifying the effectiveness of the regimen. Some HF drugs can lead the kidneys to hold on to K, as we discussed on an earlier slide but there is no mention of those drugs in this regimen so we don't need to educate on reducing K, rather, since they are on diuretics we need to watch for low K. Dig is antiarrhythmic drug, its being used here for the AFIB, it makes no sense to hold it for an irregular HR. This is a distraction option since we learned early in pharmacology class to auscultate the apical pulse for one minute and hold if less than 60 when giving dig. Patients with a heart transplantation are at risk for which of the following complications in the first year after transplantation (select all that apply)? A. Cancer B. Infection C. Rejection D. Vasculopathy E. Sudden cardiac death - CORRECT ANSWER B, C, E Cancer is not a complication of transplant. Infection risk is significant because these patients are on medications to help keep their body from rejecting the organ, this reduces their defenses to infection and can also mask early signs of infection. Rejection comes in different types and can happen early or late post transplant Vasculopathy is a complication of transplant but is not seen in the first year, this is seen later. Sudden cardiac death can occur in the event of graft failure, which tends to happen earlier post transplant Which assessment finding would the nurse expect to auscultate in a patient with mitral valve insufficiency? A. Harsh systolic murmur heard best at the right upper sternal border B. Rumbling diastolic murmur heard best at the left lower sternal border C. Holosystolic murmur heard best at the cardiac apex D. Mid-systolic click and soft diastolic murmur at the right lower sternal border - CORRECT ANSWER C Mitral insufficiency or regurgitation is characterized by a holosystolic murmur at the apex. Think back to your pneumatic devices from the previous slide Which of the following is a risk factor for a deep vein thrombosis (DVT)? A. Decreased thyroid stimulating hormone level B. Elevated liver function tests C. Hypocoagulability D. Dehydration - CORRECT ANSWER D. Dehydration A has minimal relevance, some studies have shown a potential link to two elevated liver enzyme values increasing a risk for VTE but it is not directly linked, hypocoagulapthy is a distractor to trick you into selecting it thinking it is hypercoagulability In a dehydrated state blood vessels narrow and blood becomes more concentrated, therefore creating an ideal situation for blood to pool and clot Virchow's Triad - stasis of blood flow, endothelial injury, hypercoagulability Dehydration can promote hypercoagulability Which are probably clinical findings in a patient with an acute VTE? SATA. A. Pallor and coolness of foot and calf B. Mild to moderate calf pain and tenderness C. Crossly diminished or absent pedal pulses D. Unilateral edema and induration of the thigh E. Palpable cord along a superficial varicose vein - CORRECT ANSWER B, D VTE will generally present as a warm, reddened, unilaterally swollen extremity with mild to moderate pain. A 50-year-old woman weighs 85kg and has a history of cigarette smoking, high blood pressure, high sodium intake, and sedentary lifestyle. When developing an individualized care plan for her, the nurse determines that the most important risk factors for peripheral artery disease that need to be modified are: A. Weight and diet B. Activity level and diet C. Cigarette smoking and high blood pressure D. Sedentary lifestyle and high blood pressure - CORRECT ANSWER C. Cigarette smoking and high blood pressure These really all need to be addressed but smoking most of all, especially combined with high BP A patient is admitted with peripheral artery disease. The nurse would expect to observe which of these findings in the affected lower extremity? A. Hypersensitivity to touch B. Brown pigmentation C. Pallor when elevated D. Bounding pulse - CORRECT ANSWER C. Pallor when elevated Pallor when the leg is elevated is a symptom of peripheral artery disease, compared to the brown pigmentation seen peripheral vein disease. Neither PAD or PVD will present with a bounding pulse, though pulses may be difficult to find in PAD due to poor perfusion or in PVD due to edema. Generalized pain to the touch will not be specific to either of them. In planning care and patient teaching for the patient with venous leg ulcers, the nurse recognizes that the most important intervention in healing and control of this condition is: A. Sclerotherapy B. Taking horse chestnut extract daily C. Using moist environment dressings D. Applying graduated compression stockings - CORRECT ANSWER D. Applying graduated compression stockings Compression hose will be utilized for patients with PVD but not PAD. Sclerotherapy is a treatment for varicose veins, moist dressings might be recommended but not as important as compression socks, horse extract is not widely studied or proven. "Third-spacing" can cause which form of shock? A. Anaphylactic B. Hypovolemic C. Septic D. Cardiovascular - CORRECT ANSWER B. Hypovolemic Third-spacing = movement of fluid out of the vascular system into the interstitial areas, resulting in loss of fluid leading to hypovolemic shock A patient is prescribed simvastatin (Zocor). Which of the following is an adverse effect of Zocor that should be reported immediately? A. Muscle pain B. Blurred vision C. Urinary retention D. Gastrointestinal reflux - CORRECT ANSWER A. Muscle pain Muscle pain is an adverse effect of Zocor that should be reported immediately, since muscle pain can be an indication of rhabdomyolysis and ultimately kidney failure. In a patient with thrombocytopenia, which of the following assessment findings should be immediately reported to the physician? A. Blood pressure of 210/110 B. Heart rate of 90 C. Oxygen saturation of 95% D. Specks of blood in nasal discharge after vigorous blowing - CORRECT ANSWER A. Blood pressure of 210/110 I think this question could be applicable to really any disease. A is a medical emergency regardless of the background where as the other choices are normal ranges and findings. These patients are at risk for hemorrhagic complications so elevated BP will only increase that risk. A 18-year-old male is admitted after a motor-vehicle accident with air-bag release. He complains of chest tightness and is restless and anxious. Heart sounds are barely audible on auscultation. The patient is most likely experiencing what condition? A. Cardiac tamponade B. Acute myocardial infarction C. Pulmonary edema D. Aortic aneurysm - CORRECT ANSWER A. Cardiac tamponade We know that this patient sustained trauma to the chest. Cardiac tamponade occurs when there is fluid build up around the heart that applies pressure to the heart and impairs its ability to pump adequately. This can happen with trauma or uncontrolled bleeding from an open heart procedure. Signs and symptoms include chest pain and pressure, muffled heart tones, shortness of breath, and jugular vein distention Immediately following soft tissue injury, the doctor prescribes treatment that follows the acronym "RICE," which stands for: A. Rest, immobilization, compression, exercise B. Relax, ice, conserve energy, elevation C. Rest, ice, compression, elevation D. Rigid fixator, immobilization, compression, exercise - CORRECT ANSWER C. Rest, ice, compression, elevation Added M to rice- early MOBILIZATION The nurse is preparing the plan of care for the client with an open fracture of the right arm. Which problem has the highest priority? A. Anger related to the inability to perform ADLs B. Sleep disturbances related to loss of work C. Infection related to exposed tissue D. Altered body image related to scarring - CORRECT ANSWER C. Infection related to exposed tissue When preparing the discharge teaching for the 12 year old with a fractured humerus, which information should the nurse include regarding cast care? A. Keep the arm at heart level B. Handle the cast with the tips of the fingers C. Apply an ice pack to any area that itches D. Foul smells are expected occurrences - CORRECT ANSWER A. Keep the arm at heart level The patient suffered a fractured femur. Which of the following would you tell the nursing assistant to report immediately? A. The patient complains of pain. B. The patient appears confused. C. The patient's blood pressure is 136/88. D. The patient voided using the bedpan. - CORRECT ANSWER B. The patient appears confused. Risk of FAT EMBOLISM. Will also see petechiae rash A patient with a fractured fibula is receiving skeletal traction and has skeletal pins in place. You instruct the nursing assistant to immediately report which of the following? A. The patient wants to change position in bed. B. There is a small amount of clear fluid on the pin sites. C. The traction weights are resting on the floor. D. The patient is complaining of pain and muscle spasm. - CORRECT ANSWER C. The traction weights are resting on the floor. A patient is admitted to the hospital with a possible fractured tibia. The X-ray shows that the bone is in alignment but a fracture line extends around the ankle. This type of fracture is called a: A. Comminuted fracture B. Colles' fracture C. Transverse fracture D. Greenstick fracture - CORRECT ANSWER C. Transverse fracture The nurse is preparing the preoperative client for a total hip replacement (THR). Which information should the nurse include concerning postoperative care? A. Keep abduction pillow in place between legs at all times B. Cough and deep breathe at least every 4-5 hours C. Turn to both sides every 2 hours to prevent pressure ulcers D. Sit in a high-seat chair for a flexion of less than 90 degrees - CORRECT ANSWER D. Sit in a high-seat chair for a flexion of less than 90 degrees Postoperative care should include measures to prevent dislocation of a patient's new hip prosthesis. Which of the following interventions would achieve this objective? A. Keeping the affected leg in a position of adduction B. Using pressure relief measures, other than turning, to prevent pressure ulcers C. Placing the leg in abduction D. Keeping the hip flexed by placing pillows under the patient's knee - CORRECT ANSWER C. Placing the leg in abduction As a nurse develops a postoperative nursing care plan, she knows that the patient status post total hip replacement surgery is at risk for developing complications associated with immobility. Which of the following is the most common postoperative complication for this patient? A. Pneumonia B. Thromboembolism C. Hemorrhage D. Wound infection - CORRECT ANSWER B. Thromboembolism A patient sustains a fracture of her right hip in a fall at a nursing home. Her surgery is delayed until she can be medically stabilized. Buck's traction is applied to the patient's right leg. Which of the following is true about Buck's traction? A. The head of the bed should be elevated. B. Her heel should be resting on the bed. C. The use of an overhead trapeze should be discouraged. D. The leg in traction must not be elevated on a pillow. - CORRECT ANSWER D. The leg in traction must not be elevated on a pillow. The client is diagnosed with osteoarthritis. Which sign and symptom would the nurse expect the client to exhibit? A. Severe bone deformity B. Joint stiffness C. Waddling gait D. Swan neck fingers - CORRECT ANSWER B. Joint stiffness Which foods should the nurse recommend to a client when discussing sources of dietary calcium? A. Yogurt and dark-green, leafy vegetables B. Oranges and citrus fruits C. Bananas and dried apricots D. Wheat bread and bran - CORRECT ANSWER A. Yogurt and dark-green, leafy vegetables The client is taking calcium carbonate (Tums) to help prevent further development of osteoporosis. Which teaching should the nurse implement? A. Encourage the client to take Tums with at least 8 ounces of water B. Teach the client to take Tums with the breakfast meal only C. Instruct the client to take Tums 30 to 60 minutes before meal D. Discuss the need to get a monthly serum level - CORRECT ANSWER C. Instruct the client to take Tums 30 to 60 minutes before meal You are initiating a nursing care plan for a patient with osteoporosis. All of these nursing interventions apply to the nursing diagnosis Risk for Falls. Which intervention should you delegate to the nursing assistant? A. Identify environmental factors that increase risk for falls. B. Monitor gait, balance, and fatigue level with ambulation. C. Collaborate with physical therapy to provide patient with walker. D. Assist the patient with ambulation to bathroom and in halls. - CORRECT ANSWER D. Assist the patient with ambulation to bathroom and in halls. To prevent or treat osteoporosis, adequate calcium intake: A. is essential throughout the life span B. is only necessary after menopause C. can only be obtained by supplements D. is important only until bone density peaks - CORRECT ANSWER A. is essential throughout the life span You delegate taking vital signs to an experienced nursing assistant. The patient has been diagnosed with osteomyelitis. Which vital sign do you want the nursing assistant to report immediately? A. Temperature 99.9˚F B. Blood pressure 136/80 C. Heart rate 96/minute D. Respiratory rate 24/minute - CORRECT ANSWER A. Temperature 99.9˚F An older adult client is getting out of bed for the first time. The nurse is alert for the development of which potential problem? A. Deep vein thrombosis (DVT) B. Incontinence C. Pulmonary embolism D. Orthostatic hypotension - CORRECT ANSWER D. Orthostatic hypotension The older client with cardiac disease or on antihypertensive medications is particularly at risk for orthostatic hypotension. The client would be a risk for DVT or pulmonary embolism the longer he or she remained on bed rest. Incontinence has nothing to do with getting out of bed. The nurse is working on an orthopedic unit. Which client should the nurse assess first after change of shift report? A. The 84 year-old female with a fracture right femoral neck in Buck's traction B. The 64 year-old female who had a left total knee replacement with confusion C. The 88 year-old male who had a right total knee replacement with an abduction pillow D. The 50 year-old post-op client who has a continuous passive motion (CPM) - CORRECT ANSWER B. The 64 year-old female who had a left total knee replacement with confusion What intervention will best help a client with decreased mobility decrease the risk of fractures? A. Applying a foot support B. Increasing calcium-rich foods in the diet C. Performing weight-bearing activities D. Using pressure-relieving devices - CORRECT ANSWER C. Performing weight-bearing activities Weight-bearing activity reduces bone mineral loss and promotes bone uptake of calcium, contributing to maintenance of bone density and reducing the risk for bone fractures. Although increasing the calcium in the diet is a good intervention, this alone will not reduce the client's susceptibility to bone fractures. A foot support and pressure relieving devices will not help prevent fractures, but may help with mobility and skin integrity. A client with a past history of angina has had a total knee replacement. What will the nurse teach the client prior to rehabilitation activities? A. "Use analgesics even if you are not in pain." B. "Take nitroglycerine prophylactically prior to activity." C. "Take anti-inflammatory medications before you get out of bed." D. "Do not exercise if you have knee pain." - CORRECT ANSWER B. "Take nitroglycerine prophylactically prior to activity." Participation in exercise may increase myocardial oxygen demand beyond the ability of the coronary circulation to deliver enough oxygen to meet the increased need. Nitroglycerin dilates coronary arteries within 5 minutes of use, ensuring that they will be ready to meet the demand during exercise. Which of the postoperative orders will the nurse clarify with the surgeon before discharging the client who just had arthroscopic surgery on the right knee? A. Keep right leg elevated on a soft pillow B. Non-weight-bearing by right leg for 48 hours C. Bathroom privileges with assistance and crutches D. Two tablets of Hydrocodone 10/325mg every 2 hours for pain - CORRECT ANSWER D. Max Tylenol (3000mg) A home care nurse is visiting a diabetic client with a new cast on the arm. On assessment, the nurse finds the client's fingers to be pale, cool, and slightly swollen. Which is the nurse's first intervention? a. Elevating the arm above the level of the heart b. Encouraging active and passive range of motion c. Applying heat to the affected hand d. Applying a bivalve the cast - CORRECT ANSWER a. Elevating the arm above the level of the heart Arm casts can impinge on circulation when the arm is in the dependent position. The nurse should elevate the arm above the level of the heart, ensuring that the hand is above the elbow, and reassess the extremity in 15 minutes. If the fingers are warmer and less swollen, the cast is not too tight and adjustments do not need to be made. Heat would cause more edema. Encouraging range of motion would not assist the client as much as elevating the arm. Which exercise will the nurse recommend to a client at risk for osteoporosis? a. High-impact aerobics 45 minutes once weekly b. Walking 30 minutes three times weekly c. Jogging 30 minutes four times weekly d. Bowling for 1 hour twice weekly - CORRECT ANSWER B Weight-bearing, nonjaring exercises have been proven to reduce or slow bone loss without causing vertebral compression. High-impact aerobics, jogging, and bowling are activities that actually could cause fractures in a client with osteoporosis. Which statement indicates that the client understands teaching about alendronate (Fosamax)? a. "I should take this drug with a full glass of water." b. "I need to lie down for 30 minutes after taking it." c. "This drug should be taken after a meal." d. "This drug needs to be taken at the same time with calcium." - CORRECT ANSWER A Fosamax needs to be taken on an empty stomach with a full glass of water. After taking the drug, the client needs to stay upright for 30 minutes. Fosamax should be taken on an empty stomach for best absorption. Calcium can be taken, but not at the same time as the Fosamax. While assessing an older adult client admitted 2 days ago with a fractured hip, the nurse notes that the client is confused, tachypneic, and restless. Which is the nurse's first action? a. Administering oxygen via nasal cannula b. Applying restraints c. Slowing the IV flow rate d. Discontinuing the pain medication - CORRECT ANSWER A The client is at high risk for a fat embolism and has some of the clinical manifestations. Although this is a life-threatening emergency, the nurse should take the time to administer oxygen first and then notify the health care provider. Oxygen administration can reduce the risk for cerebral damage from hypoxia. The nurse would not restrain a client who is confused without further assessment and orders. Pain medication would most likely not cause the client to be restless. The nurse notes that the skin around the client's skeletal traction pin site is swollen, red, and crusty, with dried drainage. Which is the nurse's priority intervention? a. Decreasing the traction weight b. Applying a new dressing c. Cleansing the area, scrubbing off the crusty areas d. Culturing the drainage - CORRECT ANSWER D These clinical manifestations indicate inflammation and possible infection. Infected pin sites can lead to osteomyelitis and should be treated immediately. The nurse should obtain a culture and assess vital signs. The health care provider should also be notified. Which gait-training technique is correct when teaching the client who has left leg weakness to walk with a cane? a. Placing the cane in the client's left hand and moving the cane forward, followed by moving the left leg one step forward b. Placing the cane in the client's left hand and moving the cane forward, followed by moving the right leg one step forward c. Placing the cane in the client's right hand and moving the cane forward, followed by moving the left leg one step forward d. Placing the cane in the client's right hand and moving the cane forward, followed by moving the right leg one step forward - CORRECT ANSWER c. Placing the cane in the client's right hand and moving the cane forward, followed by moving the left leg one step forward Placing the cane in the client's left hand does not provide sufficient stability. After the cane in the right hand (stronger side) is moved ahead, the cane and the stronger leg provide a stable base for movement of the weaker leg. The patient is prescribed a prick epicutaneous test to determine the cause of hypersensitivity reactions. Which result indicates the client is hypersensitive to the allergen? A. The patient complains of shortness of breath B. The skin is dry, intact, and without redness C. The pricked blood tests positive for allergens D. A pruritic wheal and erythema occurs - CORRECT ANSWER D The nurse is alerted to possible anaphylactic shock immediately after a patient has received intramuscular penicillin by the development of: A. Edema and itching at the injection site B. Sneezing and itching of the nose and eyes C. A wheal-and-flare reaction at the injection site D. Chest tightness and production of thick sputum - CORRECT ANSWER A. Edema and itching at the injection site Anaphylaxis: Commonly starts with cutaneous signs - urticaria, flushing, angioedema... (some children may develop respiratory manifestations before cutaneous) Other common s/s: rhinorrhea, dyspnea, wheezing, N/V, cardiovascular shock when severe. Which finding distinguishes rheumatoid arthritis from osteoarthritis and gouty arthritis? A. Crepitus with range of motion B. Symmetry of joint involvement C. Elevated serum uric acid levels D. Dominance in weight-bearing joints - CORRECT ANSWER B. Symmetry of joint involvement RA: mostly small joints of hands, wrists, and feet. While painful, not always red/swollen. Tends to be symmetric OA: caused by breakdown of joint cartilage and underlying bone. Most common reason for crepitus (although can be present in gout and RA) Gout: painful type of arthritis that commonly occurs in the big toe, top of the foot and ankle. Always associated with redness, swelling, and intense pain Too much Uric acid in the blood → small crystals in joints, causing inflammation and intense pain Your patient with rheumatoid arthritis (RA) is taking prednisone (Deltasone) and naproxen (Alleve) to reduce inflammation and joint pain. Which of these symptoms is the strongest indicator that a change in therapy may be necessary? A. The patient states that the RA symptoms are worst in the morning. B. The patient complains about having dry eyes. C. The patient has round and moveable nodules just under the skin. D. The patient has stools that are very dark in color. - CORRECT ANSWER D. The patient has stools that are very dark in color. Tarry stools may signify internal bleeding, possible side effect of naproxen (A) Morning stiffness usually lasts for an hour (Craven p.471) (B) Secondary Sjogren's syndrome occurs in 10%-15% of patients with RA; characterized by dry mouth and dry eye; but affects body's moisture-producing organs Which intervention has the highest priority when caring for a patient diagnosed with rheumatoid arthritis? A. Encourage the patient to ventilate feelings about the disease process. B. Discuss the effects of disease on the patient's career and other life roles. C. Instruct the patient to perform most important activities in the morning. D. Teach the patient the proper use of hot and cold therapy to provide pain relief. - CORRECT ANSWER D. Teach the patient the proper use of hot and cold therapy to provide pain relief. (C) For OA patients You assess a 24 y/o patient with RA who is considering using methotrexate for treatment. Which information is most important to communicate with the physician? A. The patient has many concerns about the safety of the drug. B. The patient has been trying to get pregnant. C. The patient takes a daily multivitamin tablet. D. The patient says that she has taken methotrexate in the past - CORRECT ANSWER (B) Methotrexate is a chemotherapeutic medication Ruth Zaiger, age 28, is admitted to the medical unit after an exacerbation of systemic lupus erythematosus (SLE). She was diagnosed with SLE 10 years ago. Ms. Zaiger is receiving intravenous methylprednisolone (Solumedrol). During short-term treatment with methylprednisolone, the nurse will most appropriately monitor the patient's: A. Weight B. liver function tests. C. serum glucose. D. respirations. - CORRECT ANSWER C. serum glucose. The nurse is preparing a female patient with SLE for discharge. Which instructions should the nurse include in the teaching plan? A. Exposure to sunlight will help control skin rashes. B. No activity limitations are necessary between flare-ups. C. Monitor body temperature. D. Corticosteroids may be stopped when symptoms are relieved. - CORRECT ANSWER C. The patient should monitor her body temperature because fever can signal an exacerbation and should be reported to the practitioner. A.Incorrect because sunlight and other sources of ultraviolet light may exacerbate the disease. B. Incorrect because fatigue can cause a flare-up of SLE, and patients should be encouraged to pace activities and plan for rest periods. Corticosteroids must be gradually tapered because they can suppress the function of the adrenal gland. Abruptly stopping corticosteroids can cause adrenal insufficiency, a potentially life-threatening situation. A patient with SLE is admitted to the hospital for evaluation and management of acute joint inflammation. Which information obtained in the admission laboratory testing concerns you most? A. Elevated blood urea nitrogen B. Increased C-reactive protein (CRP) C. Positive anti-nuclear antibody (ANA) D. Positive lupus erythematosus cell prep - CORRECT ANSWER A (A) Urea nitrogen is produced when proteins are metabolized or broken down. Elevated levels of BUN → Kidney disease, liver disease or dehydration. A possible side effect of SLE is renal failure (B) Distinguish Lupus from infection (C) Positive in 95% of patients with lupus, also may be positive with arthritis (D) Positive in 70-80% of patients with SLE The nurse is administering highly active antiretroviral therapy (HAART) to a patient with AIDS. The nurse is aware dosing of HAART medications is based on which of the following laboratory results? A. Total protein and albumin values B. Enzyme immunoassay C. HIV antibody testing D. Viral load and CD4+ counts - CORRECT ANSWER D. Viral load and CD4+ counts Patient's viral load, particular strain of virus, CD4+ cell count, and other considerations. A patient had a positive HIV antibody test 6 years ago but refused treatment due to medication side effects. The patient now has been diagnosed with AIDS and has agreed to begin antiretroviral therapy. What is the most appropriate action for the nurse to take to help the patient follow the antiretroviral treatment regimen? A. Assess the patient's routines and find adherence cues that fit into the patient's life circumstances. B. Set up a pillbox for the patient every week C. Give the patient a videotape and brochure to view at home. D. Tell the patient that the side effects of the drugs are bad but that they go away after a while. - CORRECT ANSWER A. Assess the patient's routines and find adherence cues that fit into the patient's life circumstances. In a patient who has HIV infection, the CD4+ level is measured to determine the: A. Presence of opportunistic infections B. Level of the viral load C. Extent of immune system damage D. Resistance to antigens - CORRECT ANSWER C. Extent of immune system damage (C) Determines the progression of disease progression to the immune system. CD4 count demonstrates the strength of the immune system. (A) CD4 count predicts risk for infection, not the presence of infection (B) CD4 level is independent of viral load (D) Viral load demonstrates resistance The patient diagnosed with AIDS is complaining of a sore mouth and tongue. When the nurse assesses the buccal mucosa, the nurse notes white, patchy lesions covering the hard and soft palates and the right inner cheek. Which interventions should the nurse implement? A. Teach the patient to brush the teeth and patchy area with soft-bristle tooth brush B. Notify the health care provider for an order for an anti-fungal swish-and-swallow medication C. Have the patient gargle with an antiseptic-based mouthwash several times a day D. Determine what types of food the patient has been eating for the last 24 hours - CORRECT ANSWER B. Notify the health care provider for an order for an anti-fungal swish-and-swallow medication Candida A patient with AIDS has a negative tuberculosis skin test. Which nursing action is indicated next? A. Obtain a chest X-ray and sputum smear B. No further action is needed after the negative skin test C. Teach about the anti-tuberculosis drug isoniazid (INH) D. Schedule TB testing again in 6 months - CORRECT ANSWER A. Obtain a chest X-ray and sputum smear HIV leads to increase in false positives. A patient with acute renal failure is being assessed to determine whether the cause is prerenal, intrarenal, or postrenal. If the cause is prerenal, which condition most likely caused it? A. Heart Failure B. Glomerulonephritis C. Ureterolithiasis D. Aminoglycoside toxicity - CORRECT ANSWER A. Heart Failure. By causing inadequate renal perfusion, heart failure can lead to prerenal failure. Glomerulonephritis (B) and aminoglycoside toxicity (D) are intrarenal causes, and ureterolithiasis (C) is a postrenal cause. If a patient is in the diuretic phase of acute renal failure (ARF), the nurse must monitor for which serum electrolyte imbalance? A. Hyperkalemia and hyponatremia B. Hyperkalemia and hypernatremia C. Hypokalemia and hyponatremia D. Hypokalemia and hypernatremia - CORRECT ANSWER C. Hypokalemia and hyponatremia The patient diagnosed with acute renal failure (ARF) is admitted to a medical surgical unit and placed on a therapeutic diet. Which diet would be most appropriate for this patient? A. A high-potassium and low-calcium diet B. A low-fat and low-cholesterol diet C. A high-carbohydrate and restricted-protein diet D. A regular diet with six (6) small feedings a day - CORRECT ANSWER C. A high-carbohydrate and restricted-protein diet A patient is admitted to the hospital with chronic renal failure. The nurse understands that this condition is characterized by: A. Progressive irreversible destruction of the kidneys B. A rapid decrease in urinary output with an elevated BUN C. An increasing creatinine clearance with a decrease in urinary output D. Prostration, somnolence, and confusion with coma and imminent death - CORRECT ANSWER A. Progressive irreversible destruction of the kidneys The patient diagnosed with ESRD is receiving peritoneal dialysis. Which assessment data warrants immediate intervention by the nurse? A. Inability to auscultate a bruit over the fistula B. The client's abdomen is soft, is nontender, and has bowel sounds C. The dialysate being removed from the client's abdomen is clear D. The dialysate instilled into the client was 1500 mL and that removed was 1500 mL - CORRECT ANSWER D Because the patient is in ESRD, fluid must be removed from the body so the output should be more than the amount instilled. These assessment data require intervention by the nurse. (A) Peritoneal dialysis is administered through a catheter inserted into the peritoneal cavity; a fistula is used for hemodialysis. (B) Peritonitis, inflammation of the peritoneum, is a serious complication that would result in a hard, rigid abdomen. Therefore, a soft abdomen would not warrant immediate intervention. (C) The dialysate return is normally colorless or straw-colored, but it should never be cloudy, which indicates an infection. Which observation involving a patient's fistula would require the nurse to notify the doctor? A. Blood flow detected while palpating the fistula site B. Blood flow observed through the cannula C. Absence of an audible bruit while auscultating the graft D. Straw-colored blood flow observed through the cannula - CORRECT ANSWER C The nurse should hear turbulent blood flow through the vessels using the bell of the stethoscope; absent bruit indicates a non-patent fistula, requiring the nurse to notify the doctor. Blood flow detected while palpating the fistula site indicates that the fistula is patent; notifying the doctor wouldn't be necessary. Because an arteriovenous fistula doesn't require an external cannula, blood flow - regardless of color, wouldn't be visible. In chronic renal failure, symptoms may not become apparent until later stages of the disease because: A. Liver hormones mask the symptoms B. The kidneys have a great functional reserve C. Other body systems take over some of the kidney's functions D. The adrenal glands compensate for the kidney's decreased function - CORRECT ANSWER B. The kidneys have a great functional reserve Because of the great functional reserve of the kidneys, chronic renal failure develops more slowly than acute renal failure and signs and symptoms don't appear until later stages of the disease. Liver hormones don't mask symptoms of renal failure, and other body systems don't compensate for the kidney's decreased function. A patient is admitted to the hospital with a history of chronic renal failure. Which of these laboratory tests is the most accurate indicator of a patient's renal function? A. Blood urea nitrogen B. Creatinine clearance C. Serum creatinine D. Urinalysis - CORRECT ANSWER B. Creatinine clearance Creatinine clearance closely correlates with the kidney's glomerular filtration rate and tubular excretion ability. Results from blood urea nitrogen, serum creatinine, and urinalysis may be influenced by various conditions and aren't specific to renal disease, such as dehydration. The immunologic mechanism involved in glomerulonephritis include: A. Tubular blocking by precipitates of bacteria and antibody reactions B. Deposition of immune complexes and complement along the GBM C. Thickening of the GBM from autoimmune microangiopathic changes D. Destruction of glomeruli by proteolytic enzymes contained in the GBM - CORRECT ANSWER B. Deposition of immune complexes and complement along the GBM One of the most important roles of the nurse in relation to acute post-streptococcal glomerulonephritis is to: A. Promote early diagnosis and treatment of sore throats and skin lesions B. Encourage patients to request antibiotic therapy for all upper respiratory infections C. Teach patients with APSGN that long-term prophylactic antibiotic therapy is necessary to prevent recurrence D. Monitor patients for respiratory symptoms that indicate that the disease is affecting the alveolar basement membrane - CORRECT ANSWER A. Promote early diagnosis and treatment of sore throats and skin lesions The elderly patient is diagnosed with chronic glomerulonephritis. Which lab value indicates the condition has gotten worse? A. The BUN is 15 mg/dL. B. The creatinine level is 1.2 mg/dL. C. The glomerular filtration rate is 40 mL/minute. D. The 24-hour creatinine clearance is 100 mL/minute. - CORRECT ANSWER C. The glomerular filtration rate is 40 mL/minute. Glomerular filtration rate (GFR) is approximately 120 mL per minute. If the DFR is decreased to 40 mL per minute, the kidneys are functioning at about 1/3 filtration capacity. A.Normal blood urea nitrogen levels are 7-18 mg/dL or 8-20 mg/dL for clients older than age 60 years. B.Normal creatinine levels are 0.6 - 1.2 mg/dL. D. Normal creatinine clearance is 85-125 mL per minute for males and 75-115 mL per minute for females. Test taking hint: One must memorize common lab values. BUN and creatinine levels are common lab values used to determine status in a # of diseases. Answers A and B are normal values and could be eliminated. Then one would have to choose from only 2 options. You are the admission nurse for a patient with nephrotic syndrome. Which assessment finding supports this diagnosis? A. Edema formation B. Hypotension C. Increased urine output D. Flank pain - CORRECT ANSWER A. Edema formation The underlying pathophysiology of nephrotic syndrome involves increased glomerular permeability that allows larger molecules to pass through the membrane into the urine and be removed from the blood. This process causes massive loss of protein, edema formation, and decreased serum albumin levels. Key features include hypertension and renal insufficiency (decreased urine output). Flank pain is seen in patients with acute pyelonephritis. A patient is admitted to the hospital with severe renal colic caused by renal lithiasis. The nurse's first priority in management of the patient is to: A. Administer opioids as prescribed B. Obtain supplies for straining all urine C. Encourage fluid intake of 3 to 4 L/day D. Keep the patient NPO in preparation for surgery - CORRECT ANSWER A. Administer opioids as prescribed A patient with a ureterolithotomy returns from surgery with a nephrostomy tube in place. Postoperative nursing care of the patient includes: A. Encouraging the patient to drink fruit juices and milk B. Forcing fluids of at least 2-3 L per day after nausea has subsided C. Irrigating the nephrostomy tube with 10 mL of normal saline solution as needed D. Notifying the physician if nephrostomy tube drainage is more than 30 mL per hour - CORRECT ANSWER B. Forcing fluids of at least 2-3 L per day after nausea has subsided The nurse teaches the female patient who has frequent UTIs that she should: A. Take tub baths with bubble bath B. Urinate before and after sexual intercourse C. Take prophylactic sulfonamides for the rest of her life D. Restrict fluid intake to prevent the need for frequent voiding - CORRECT ANSWER B. Urinate before and after sexual intercourse In teaching a patient about the disorder, the nurse informs the patient that the organisms that cause pyelonephritis most commonly reach the kidneys through: A. The bloodstream B. The lymphatic system C. A descending infection D. An ascending infection - CORRECT ANSWER D. An ascending infection According to the American Cancer Society, fecal occult blood testing should be done annually after the age of _______ years. 1. 30 2. 40 3. 50 4. 60 - CORRECT ANSWER 3. 50 Fecal occult blood testing for colorectal cancer should be done annually for both men and women after the age of 50 years. Teat-Taking Strategy: To answer this question correctly, you must be familiar with the recommendations for cancer screening published by the American Cancer Society. This would allow you to eliminate each of the incorrect options easily. The ambulatory care nurse measures blood pressure of a client and finds it to be 156/94 mmHg. Which of the following areas is unnecessary to emphasize when providing client education for blood pressure control? 1. Instruct the client to limit protein intake. 2. Teach the client to avoid adding salt to foods. 3. Discuss the rationale for reducing or maintaining weight. 4. Stress the importance of a regular exercise program. - CORRECT ANSWER 1. Instruct the client to limit protein intake. Obesity and sodium are modifiable risk factors for hypertension. These are of the utmost importance because they can be changed or modified by the individual through a regular exercise program and careful monitoring of sodium intake. Protein intake has no relationship to hypertension. Test-Taking Strategy: Focus on the strategic words "unnecessary to emphasize" in the question. This will assist in directing you to instruct the client to limit protein intake. Review the client education for risk factors reduction for hypertension if you had difficulty with this question. Which of the following teaching points is the priority when the nurse is teaching the client about caring for a plaster cast? 1. The cast gives off heat as it dries. 2. The client can bear weight on the cast in 1 hour. 3. A stockinette and soft padding are put over the leg area before casting. 4. Immediately report any increase in drainage or interruption in cast integrity. - CORRECT ANSWER 4. Immediately report any increase in drainage or interruption in cast integrity. Increases in drainage or interruption in cast integrity will affect healing and could lead to osteomyelitis. To apply a cast, the skin is washed and dried well. A stockinette is placed smoothly and evenly over the area to be casted, followed by a roll of padding. The plaster is then rolled onto the padding, and the edges are trimmed or smoothed if needed. A plaster cast gives off heat as it dries. A plaster cast can tolerate the weight-bearing once it is dry, which varies from 24 to 72 hours, depending on the nature and thickness of the cast. Test-Taking Strategy: Note the word "priority." Recalling the drainage is a sign of infection will direct you to "immediately report any increase in drainage or interruption in cast integrity." Review the principles of cast care and the client teaching points if you had difficulty with this question. The nurse is providing instructions to a client and family regarding home care following cataract removal from the left eye. The nurse would plan to teach the client which of the following pieces of information about positioning in the postoperative period? 1. Lower the head between the knees 3 times a day. 2. Bend below the waist as often as possible. 3. Avoid sleeping on the left side. 4. Sleep only on the left side. - CORRECT ANSWER 3. Avoid sleeping on the left side. Following cataract surgery, the client should not sleep on the left side of he body that was operated on. Clients should also avoid bending below the level of the waist so as not to increase intraocular pressure. Test-Taking Strategy: Remember that options that are comparable or alike are not likely to be correct. With this in mind, eliminate "lower the head between the knees 3 times a day" and "bend below the waist as often as possible" first. Choose correctly between the two remaining options, using knowledge of positioning following eye surgery and principles related to gravity and edema formation. Review the care to the client following cataract surgery if you had difficulty with this question. The client with hypertension has been prescribed a low-sodium diet. The nurse teaching this client about foods that are allowed should plan to include which of the following in a list provided to the client? 1. Tomato soup 2. Boiled shrimp 3. Instant oatmeal 4. Summer squash - CORRECT ANSWER 4. Summer squash Foods that are lower in sodium are fruits and vegetables (summer squash) because they do not contain physiological saline. Highly processed or refined foods ( tomato soup and instant oatmeal) are higher in sodium unless they are specifically noted as "low sodium." Saltwater fish and shellfish are higher in sodium. Test-Taking Strategy: Begin to answer this question by eliminating "boiled shrimp" as being highest in sodium. Eliminate the other correct options they are processed foods. Review the foods that are high and low in sodium if you had difficulty with this question. The client has been diagnosed with gout. In developing a teaching plan for this client, the nurse should include a list that identifies which of the following foods to be avoided? 1. Chicken liver 2. Carrots 3. Tapioca 4. Chocolate - CORRECT ANSWER 1. Chicken liver Liver and other organ meats should be omitted from the diet of a client who has gout because of high purine content. Purines are a form of protein. The food items identified in the other options contain negligible amounts of purines and may be consumed freely by the client with gout. Test-Taking Strategy: Focus on the pathophysiology of the client's diagnosis and subject, foods high in purine. Remember that organ meats are high in purines. The client is resuming a diet after hemigastrectomy. To minimize complications, the nurse would tell the client to avoid doing which of the following? 1. Lying down after eating 2. Drinking liquids with meals 3. Eating six meals a day 4. Excluding concentrated sweets in the diet - CORRECT ANSWER 2. Drinking liquids with meals The client who had a hemigastrectomy is at risk for dumping syndrome. This client should be placed on a diet that is high in protein, moderate in fat, and high in calories. The client should avoid drinking liquids with meals. Frequent small meals are encouraged, and the client should avoid concentrated sweets. The nurse is providing dietary teaching to a client receiving potassium-sparing diuretic about foods that are low in potassium. The nurse should include which of the following on a list of foods that have low potassium content? 1. Apple 2. Carrots 3. Spinach 4. Avocado - CORRECT ANSWER 1. Apple On medium apple with skin provides approximately 159 mg of potassium per serving, so it has the lowest potassium content of these choices. Raw spinach (oz) provides 470 mg of potassium. One large carrot has 341 mg of potassium. One medium avocado provides the highest potassium content, 1097 mg. Test-Taking Strategy: Focus on the subject, the low potassium food. Use knowledge about the nutritional content of foods to assist in directing you to "apple." The nurse understands that increasing the flow of oxygen to more than 2L/min in the client with chronic obstructive pulmonary disease (COPD) could be harmful because it: 1. Is drying the nasal mucosal passages 2. Decreases diaphragmatic excursion and depth 3. Increases the risk of pneumonia and atelectasis 4. Decreases the client's oxygen-based respiratory drive - CORRECT ANSWER 4. Decreases the client's oxygen-based respiratory drive Normally, respiratory rate varies with the amount of carbon dioxide present in the blood. In clients with COPD, this natural process becomes ineffective after exposure to high levels of carbon dioxide for prolonged periods of time. Instead, the level of oxygen provides the respiratory stimulus. The client with COPD cannot increase oxygen levels independently because this could decrease the respiratory drive, leading to respiratory failure. Test-Taking Strategy: Use knowledge of basic respiratory physiology to answer this question. Remember that the client with COPD should not increase the flow of oxygen to more than 2L/min, because this could severely decrease the respiratory drive. The nurse is caring for a client with chronic obstructive pulmonary disease (COPD) anticipates which of the following arterial blood gases (ABG) findings? 1. pH, 7.40; PaO2 90 mmHg; CO2 39 Meq/L; HCO3, 23 mEq/L 2. pH, 7.32; PaO2,85 mmHg; CO2 57 Meq/L; HCO3, 26 mEq/L 3. pH, 7.47; PaO2 82 mmHg; CO2 30 Meq/L; HCO3, 31 mEq/L 4. pH, 7.31; PaO2 95 mmHg; CO2 22 Meq/L; HCO3, 19mEq/L - CORRECT ANSWER 2. pH, 7.32; PaO2,85 mmHg; CO2 57 Meq/L; HCO3, 26 mEq/L The client with COPD will exist is a state of respiratory acidosis - pH, 7.40; PaO2 90 mmHg; CO2 39 Meq/L; HCO3, 23 mEq/L and increased CO2 whioch acts as as acid in the body. It is elevated in the client with COPD because of the inability to exhale well and eliminate CO2. Therefore with a rise in CO2, there is a corresponding fall in pH. The answer with pH, 7.31; PaO2 95 mmHg; CO2 22 mEq/L; HCO3, 19mEq/L reflect an acidotic pH but the CO2 is normal. Test-Taking Strategy: Remember that a client with COPD will exist in a state of respiratory acidosis. Also recall that there is an inverse relationship between the pH and the CO2 in the body. As CO2 rises, pH falls; as CO2 falls, pH rises. This concept forms the basis for primary concepts of acid-base balance and respiratory disorders. NORMAL pH: acidotic...7.35 - 7.45...alkalotic CO2: 35 - 45 HCO3: acidotic...22 - 26...alkalotic The client with COPD is experiencing exacerbation of the disease. The nurse would determine that which of the following documented in the client's record is an expected finding with this client? 1. Increased oxygen saturation with ambulation 2. Hyperinflation of lungs documented by chest x-ray 3. A widened diaphragm documented by chest x-ray 4. A shortened expiratory phase of the respiratory cycle - CORRECT ANSWER 2. Hyperinflation of lungs documented by chest x-ray The clinical manifestations of COPD are several including hypoxemia, hypercapnia, dyspnea on exertion and at rest; oxygen desaturation with exercise; use of accessory muscles; and prolonged exhalation. Chest x-ray results indicate a hyperinflated chest and may indicate a flattened diaphragm if the disease is advanced. Test-Taking Strategy: Use of process of elimination after reading each option carefully. Eliminate "increased oxygen saturation with ambulation" because the revers is true, even in the client with no respiratory disorder. Eliminate "a widened diaphragm documented by chest x-ray' because it is the opposite of what happens with exacerbation. Eliminate "a shortened expiratory phase of the respiratory cycle" because the client with COPD has a prolonged expiratory phase. Review the clinical manifestations of COPD if you had difficulty with this question. A 30 year-old man with chronic traumatic brain injury was admitted for a surgical procedure and is to be discharged. The nurse should modify his discharge instructions by: 1. Providing detailed oral instructions 2. Providing detailed written instructions 3. Having him repeat the instructions 4. Having sustained eye contact during the instruction - CORRECT ANSWER 3. Having him repeat the instructions John Bouknight, age 62, has a five-year history of Parkinson's Disease (PD). As Mr. Bouknight ambulates with a walker, he stops and tells the nurse his legs "feel as is they are glued to the floor." The nurse recognizes this as the "freezing of gait," a common development in PD that requires the nurse to: 1. Report the finding to the physical therapist. 2. Initiate fall risk procedures for the patient 3. Call the physician to discuss medication side effects 4. Initiate relaxation training to assist the patient with ambulation. - CORRECT ANSWER 2. Initiate fall risk procedures for the patient The nurse observes Mr. Bouknight becoming agitated and non-directable, picking at things in the air. The nurse identifies this as: 1. a usual symptom in PD and secludes the patient in a quiet area. 2. most likely due to overdosing of levodopa (Sinemet) and calls the physician. 3. indicative of a seizure and lowers the patient to the floor. 4. suggestive of dehydration and provides additional oral fluids. - CORRECT ANSWER 2. most likely due to overdosing of levodopa (Sinemet) and calls the physician. In talking with a nursing orientee about Mr. Bouknight's diagnosis, the nurse identifies which of the following as the most frequent cause of death in the patients with PD? 1. pneumonia 2. injuries from falls 3. injuries other than falls 4. venous thromboembolism - CORRECT ANSWER 1. pneumonia RESPIRATORY is always the answer The nurse is assessing a male patient with gonorrhea. Which symptom most likely prompted him to seek medical treatment? A. Rashes on the palms of the hands and soles of the feet B. Cauliflower-like warts on the penis C. Painful red papules on the shaft of the penis D. Foul-smelling discharge from the penis - CORRECT ANSWER D. Foul-smelling discharge from the penis You obtain all of these assessment data about your client with continuous bladder irrigation (CBI) after a transurethral resection of the prostate (TURP). Which information indicates the most immediate need for nursing interventions? A. The client states he feels a continuous urge to void. B. The catheter drainage is light pink with occasional clots. C. The catheter is pulled semi-taut and taped to the client's thigh. D. The client complains of painful bladder spasms - CORRECT ANSWER D. The client complains of painful bladder spasms Slow down the rate, resume rate as soon as bladder spasms subside Which data would indicate that discharge teaching has been effective for the client who is post operative transurethral resection of the prostate (TURP)? A. "I will call the surgeon if I experience any difficulty urinating." B. "I will take my Proscar daily, the same as before my surgery." C. "I will continue restricting my oral fluid restrictions." D. "I will need to take my pain medication routinely even if I do not hurt." - CORRECT ANSWER A. "I will call the surgeon if I experience any difficulty urinating." What is the American Cancer Society's recommendation for the early detection of cancer of the prostate? A. A yearly PSA level and DRE beginning at age 50 B. A biannual rectal examination beginning at age 40 C. A semi-annual alkaline phosphatase level beginning at 45 D. A yearly urinalysis to determine the presence of prostatic fluid - CORRECT ANSWER A. A yearly PSA level and DRE beginning at age 50 DRE= Digital Rectal Examination The nurse is speaking to a group of women about early detection of breast cancer. The average age of the group is 47. Following the American Cancer Society guidelines, the nurse should recommend that the women: A. Perform breast self examination (BSE) annually B. Have a mammogram annually C. Have a hormone receptor assay annually D. Have a healthcare provider conduct a clinical examination every 2 years - CORRECT ANSWER B. Have a mammogram annually You observe a student nurse accomplishing all of these activities while caring for a client who has an intracavity radioactive implant in place to treat cervical cancer. Which action requires that you intervene immediately? A. The student stands next to the client for 5 minutes while assisting with her bath. B. The student asks the client how she feels about losing her child-bearing ability. C. The student assists the client to the bedside commode for a bowel movement. D. The student offers to get the client whatever she would like to eat or drink. - CORRECT ANSWER C. The student assists the client to the bedside commode for a bowel movement. No bm for 3 days if not radioactive material will come out Which patient has the highest risk of ovarian cancer? A. A 30 year-old woman taking an oral contraceptive B. A 45 year-old woman who has never been pregnant C. A 40 year-old woman with children D. A 36 year-old woman who had her first child at age 22 - CORRECT ANSWER B. A 45 year-old woman who has never been pregnant The 24 year-old female client presents to the clinic with lower abdominal pain on the left side that she rates a "9" on a 1-10 scale. Which diagnostic procedure should the nurse prepare the client for? A .A computed tomography scan B. A lumbar puncture C. An appendectomy D. A pelvic sonogram - CORRECT ANSWER D. A pelvic sonogram A patient is admitted to the hospital with shaking chills and fever. His red blood cell count is abnormally low, causing the doctor to suspect aplastic anemia. The patient is at greater risk for infection with aplastic anemia because: A. A large amount of immature red blood cells (RBCs) are produced. B. Granulocyte production is depressed. C. An excessive number of thrombocytes are present. D. Vitamin B12 isn't being absorbed by the stomach. - CORRECT ANSWER B. Granulocyte production is depressed. Aplastic anemia results when the bone marrow produces insufficient quantities of RBCs, white blood cells, and platelets. Granulocytes, a type of white blood cell, are needed to fight infection. (A) The quantity of blood cells is affected, but not the quality; immature RBCs aren't produced. (C) Thrombocyte production affects blood clotting, and vitamin B12 deficiency leads to pernicious anemia. Which of the following drugs that a patient has taken may be responsible for causing aplastic anemia? A. Ibuprofen (Motrin) B. Sulfamethoxazole-trimethoprim (Bactrim) C. Levothyroxine (Synthroid) D. Heparin (Liquaeim) - CORRECT ANSWER B. Sulfamethoxazole-trimethoprim (Bactrim) Sulfa drugs such as sulfamethoxazole-trimethoprim are occasionally linked to aplastic anemia. (A, C, D) Ibuprofen, levothyroxine, and heparin don't cause aplastic anemia. When assessing a patient with anemia from blood loss, the nurse would expect to find which of the following symptoms? A. Sudden onset of symptoms, hypotension and tachycardia B. Exertional dyspnea, poor nutrition and hypotension C. Sudden onset of symptoms, glossitis and tachycardia D. Fatigue, neuropathy and tachycardia - CORRECT ANSWER A. Sudden onset of symptoms, hypotension and tachycardia Acute blood loss occurs suddenly. Hypotension and tachycardia are compensatory mechanisms in response to rapid loss of blood volume. (B) Although anemia may produce exertional dyspnea and hypotension, answer B is incorrect because poor nutrition is neither a symptom nor a cause of anemia due to blood loss. (C, D) are incorrect because anemia due to acute blood loss neither produces glossitis nor neuropathy. A client admitted to the hospital with a sickle cell crisis complains of severe abdominal, hip, and knee pain. You observe an LVN accomplishing these client care tasks. Which one requires that you, as charge nurse, intervene immediately? A. The LVN encourages the client to use the ordered PCA. B. The LVN positions cold packs on the client's knees. C. The LVN places a "No Visitors" sign on the client's door. D. The LVN checks the client's temperature every 2 hours. - CORRECT ANSWER B. The LVN positions cold packs on the client's knees. The joint pain that occurs in sickle cell crisis is caused by obstruction of blood flow by the sickled red blood cells. The appropriate therapy for this client would be application of moist heat to the joints to cause vasodilation and improve circulation. (A) Because control of pain is a priority during sickle cell crisis, encouraging the client to use the PCA is an appropriate therapy. (C, D) While infection control is important in preventing and treating sickle cell crisis, there is no need to restrict all visitors or to check the temperature every 2 hours. The nurse is transcribing the HCP's order for an iron supplement on the MAR. At which time should the nurse schedule the daily dose? A. 0900 B. 1000 C. 1200 D. 1630 - CORRECT ANSWER B. 1000 This is approximately two (2) hours after breakfast and is the correct dosing time for iron to achieve the best effects. Iron preparations should be administered one (1) hour before a meal or two (2) hours after a meal. Iron can cause gastrointestinal upset, but if administered with a meal, absorption can be diminished by as much as 50%. (A) The usual medication dosing time for daily medications is 0900, but this is only an hour after the breakfast meal. Iron absorption is reduced when taken with food. (C) This is the usual time that health-care facilities serve lunch. (D) This time would be very close to the evening meal and would decrease the absorption of the iron. The nurse is discharging a client diagnosed with anemia. Which discharge instruction should the nurse teach? A. Take the prescribed iron until it is completely gone. B. Monitor pulse and blood pressure at a local pharmacy weekly. C. Have a complete blood count checked at the HCP's office. D. Perform isometric exercise three (3) times a week. - CORRECT ANSWER C. Have a complete blood count checked at the HCP's office. The client should have a complete blood count regularly to determine the status of the anemia. (A) This is an instruction for antibiotics, not iron. The client will take iron for an indefinite period. (B) Pulse is indirectly affected by anemia when the body attempts to compensate for the lack of oxygen supply, but this is an indirect measure, and blood pressure is not monitored for anemia. (D) Isometric exercises are bodybuilding exercises, and the client should not be exerting himself or herself in this matter. The nurse writes a client problem of "activity intolerance" for a client diagnosed with anemia. Which intervention should the nurse implement? A. Pace activities according to tolerance. B. Provide supplements high in iron and vitamins. C. Administer packed red blood cells. D. Monitor vital signs every four (4) hours. - CORRECT ANSWER A. Pace activities according to tolerance. The client's problem is activity intolerance, and pacing activities directly affect the diagnosis. (B) This is an appropriate intervention for iron or vitamin deficiency, but it is not for activity intolerance. (C) This may be done, but not specifically for the diagnosis. (D) This would not help activity intolerance. The client diagnosed with thalassemia, a hereditary anemia, is to receive a transfusion of packed RBCs. The cross-match reveals the presence of antibodies that cannot be cross-matched. Which precaution should the nurse implement when initiating the transfusion? A. Start the transfusion at 10-15 mL per hour for 15-30 minutes. B. Re-crossmatch the blood until the antibodies are identified. C. Have the client sign a permit to receive uncrossmatched blood. D. Have the licensed nursing assistant stay with the client. - CORRECT ANSWER A. Start the transfusion at 10-15 mL per hour for 15-30 minutes. It can be difficult to cross-match blood when antibodies are present. If imperfectly cross-matched blood must be transfused, the nurse must start the blood very slowly and stay with the client, monitoring frequently for signs of a hemolytic reaction. (B) The antibodies have been identified. The donor blood does not cross-match perfectly to the blood of the client. (C) The blood has been cross-matched. Client permits regarding uncross-matched blood are used for emergency transfusions when time does not allow an attempt to cross-match. In such a case, O-blood, the universal donor, will be used. (D) The nurse cannot delegate an unstable client to an unlicensed nursing assistant. The nurse must stay with the client. A client with acute myelogenous leukemia is receiving induction phase chemotherapy. Which assessment information is of most concern? A. Serum potassium level of 7.8 mEq/L B. Urine output less than intake by 400 mL C. Inflammation and redness of oral mucosa D. Ecchymoses present on anterior trunk - CORRECT ANSWER You are transferring a client with newly diagnosed chronic myeloid leukemia to a long-term-care (LTC) facility. Which information is most important to communicate to the LTC charge nurse prior to transferring the client? A. The Philadelphia chromosome is present in the blood smear. B. Glucose is elevated as a result of prednisone therapy. C. There has been a 20-pound weight loss over the last year. D. The client's chemotherapy has resulted in neutropenia. - CORRECT ANSWER D. The client's chemotherapy has resulted in neutropenia. The neutropenic client is at increased risk for infection, so the LTC charge nurse needs to know this in order to make decisions about the client room assignment and to plan care (A, B, C) The other information also will impact on planning for client care, but the charge nurse needs the information about neutropenia before the client is transferred. A long-term-care client with chronic lymphocytic leukemia has a nursing diagnosis of Activity Intolerance related to weakness and anemia. Which of these nursing activities is most appropriate for you, as the charge nurse, to delegate to a nursing assistant? A. Evaluate the client's response to normal activities of daily living. B. Check the client's blood pressure and pulse rate after ambulation. C. Determine which self-care activities the client can do independently. D. Assist the client in choosing a diet that will improve strength. - CORRECT ANSWER B. Check the client's blood pressure and pulse rate after ambulation. Nursing assistant education includes routine nursing skills such as assessment of vital signs. (A, C, D) Evaluation, baseline assessment of client abilities, and nutrition planning are roles appropriate to RN practice. When teaching safety precautions to a patient with thrombocytopenia, the nurse should include which of the following directives? A. Eat foods high in iron B. Avoid products that contain aspirin C. Avoid people with respiratory tract infections D. Eat only cooked vegetables - CORRECT ANSWER B. Avoid products that contain aspirin Patients with a low platelet count should avoid products that contain aspirin because they increase the tendency to bleed. (A) would be important to teach the patient with anemia. (C, D) are correct for the patient with leukopenia. The nurse is admitting a client with rule-out Hodgkin's lymphoma. When the nurse assesses the client, which data would support this diagnosis? A. Night sweats and fever without "chills." B. Edematous lymph nodes in the groin. C. Malaise and complaints of an upset stomach. D. Pain in the neck area after a fatty meal. - CORRECT ANSWER A. Night sweats and fever without "chills." Clients in late stages of Hodgkin's disease experience drenching diaphoresis, especially at night; fever without chills; and unintentional weight loss. Early-stage disease is indicated by a painless enlargement of a lymph node on one side of the neck (cervical area). Pruritus is also a common symptom. (B) Lymph node enlargement with Hodgkin's disease is in the neck area. (C) Malaise and stomach complaints are not associated with Hodgkin's disease. (D) Pain in the neck area at the site of the cancer occurs in some clients after the ingestion of alcohol. The cause for this is unknown. Which test is considered diagnostic for Hodgkin's lymphoma? A. A magnetic resonance image (MRI) of the chest. B. A computed tomography (CT) scan of the cervical area. C. An erythrocyte sedimentation rate (ESR) D. A biopsy of the cervical lymph nodes. - CORRECT ANSWER D. A biopsy of the cervical lymph nodes. Cancers of all types are definitely diagnosed through biopsy procedures. The pathologist through biopsy procedures. The pathologist must identify Reed-Sternberg cells for a diagnosis of Hodgkin's disease. (A) An MRI of the chest area will determine numerous disease entities, but it cannot determine the specific morphology of Reed-Sternberg cells, which are diagnostic for Hodgkin's disease. (B) A CT scan will show tumor masses in the area, but it is not capable of pathological diagnosis. (C) ESR lab tests are sometimes used to monitor the progress of the treatment of Hodgkin's disease, but ESR levels can be elevated in several disease processes. The nurse is reviewing assessment data for a patient with a diagnosis of Stage III Hodgkin's disease. This diagnosis is most strongly supported by lymphatic involvement on both sides of the: A. Blood brain barrier B. Diaphragm C. Descending aorta D. Spinal column - CORRECT ANSWER B. Diaphragm With stage III Hodgkin's disease, malignant cells are widely disseminated to lymph nodes on both sides of the diaphragm. (A, B, D) are incorrect because these structures aren't involved in the staging of Hodgkin's disease. Which clinical manifestation of Stage I non-Hodgkin's lymphoma would the nurse expect to find when assessing the client? A. Enlarged lymph tissue anywhere in the body. B. Tender left upper quadrant. C. No symptom in this stage. D. Elevated B cell lymphocytes on the CBC. - CORRECT ANSWER C. No symptom in this stage. Stage I lymphoma presents with no symptoms; for this reason, clients are usually not diagnosed until the later stages of lymphoma. (A) Enlarged lymph tissue would occur in stage III or IV Hodgkin's lymphoma. (B) A tender left upper quadrant would indicate spleen infiltration and occur at a later stage. (D) B cell lymphocytes are the usual lymphocytes that are involved in the development of lymphoma, but a serum blood test must be done specifically to detect B cells. They are not tested on a CBC. The 33 year-old client diagnosed with Stage IV Hodgkin's lymphoma is at the five (5)-year remission mark. Which information should the nurse teach the client? A. Instruct the client to continue scheduled screenings for cancer. B. Discuss the need for follow-up appointments every five (5) years. C. Teach the client that the cancer risk is now the same as for the general population. D. Have the client talk with the family about funeral arrangements. - CORRECT ANSWER A. Instruct the client to continue scheduled screenings for cancer. The five (5) year mark is a time for celebration for clients diagnosed with cancer, but the therapies can cause secondary malignancies and there may be a genetic predisposition for the client to develop cancer. The client should continue to be tested regularly. (B) Follow-up appointments should be at least yearly. (C) The client's risk for developing cancer has increased as a result of the therapies undergone for the lymphoma. (D) This client is in remission, and death is not imminent. You obtain the following data about a client admitted with multiple myeloma. Which information has the most immediate implications for the client's care? A. The client complains of chronic bone pain. B. The blood uric acid level is very elevated. C. The 24-hour urine shows Bence-Jones protein. D. The client is unable to plantar flex the feet. - CORRECT ANSWER D. The client is unable to plantarflex the feet. The lack of plantar flexion may indicate spinal cord compression, which should be evaluated and treated immediately by the physician to prevent further loss of function. (A, B, C) While chronic bone pain, hyperuricemia, and the presence of Bence-Jones protein in the urine are all typical of multiple myeloma and do require assessment and/or treatment, the loss of motor or sensory function is an emergency. Which is the least important area of assessment for the patient with multiple myeloma? A. Skeletal system B. Renal system C. Nervous system D. Cardiovascular system - CORRECT ANSWER D. Cardiovascular system Multiple myeloma usually don't have a direct effect on the heart. (A, B, C) are incorrect because multiple myeloma usually affects the skeletal, renal, and nervous systems. After receiving change-of-shift report about all of these clients, which one will you assess first? A. A 26 year-old with thalassemia major who has a short-stay admission for a blood transfusion. B. A 44 year-old who was admitted 3 days previously with a sickle cell crisis and has orders for a CT scan. C. A 50 year-old with newly diagnosed stage IV non-Hodkin's lymphoma who is crying and stating "I'm not ready to die" D. A 69 year-old with chemotherapy-induced neutropenia who has an elevated oral temperature. - CORRECT ANSWER D. A 69 year-old with chemotherapy-induced neutropenia who has an elevated oral temperature. The nurse is teaching the patient and family about managing peptic ulcer disease. The nurse should include which of the following as part of dietary instruction? a. Take caffeine-containing beverages in moderation b. Eat large meals to alleviate disease symptoms. c. Have a bedtime snack every evening. d. Eat quickly to prevent over distention and reflux. - CORRECT ANSWER a. Take caffeine-containing beverages in moderation Caffeine-containing beverages (coffee, tea, cola drinks) and decaffeinated coffee cause increased gastric acid production but may be taken in moderation at or near mealtime if tolerated. The patient should eat three small meals and three snacks evenly spaced through the day to avoid periods of hunger or overeating. Foods should be eaten slowly and chewed well. The patient should avoid eating within 3 hours before bedtime; bedtime snacks can cause gastric acid secretion during the night. A patient has a nasogastric (NG) tube to low constant suction following a partial gastrectomy with a vagotomy. Following standards of practice, the nurse would question the surgeon about which of the following postoperative orders: a. Notify physician for temperature over 101.5 degrees F. b. Call physician for any bright red blood from the NG tube. c. Irrigate the NG tube with 30 ml normal saline every 8 hours. d. Keep the NG tube taped and secured to the patient's nares. - CORRECT ANSWER c. Irrigate the NG tube with 30 ml normal saline every 8 hours. The nurse should not irrigate or move the NG tube to avoid disrupting the internal suture line of the stomach. All other options are generally expected postoperative care measures. A patient with a new diagnosis of erosive esophagitis has been prescribed esomeprazole (Nexium). The nurse should teach the patient to: a. take the medication an hour before meals. b. avoid taking antacids with this medication. c. mix and take within 60 minutes, if using a suspension. d. crush caplets and mix with applesauce if swallowing is difficult - CORRECT ANSWER a. take the medication an hour before meals. Esomeprazole should be taken an hour before meals. Antacids can be taken with this medication. IF the patient is using the suspension, it must be taken within 30 minutes or discarded. Esomeprazole is also available in capsules that can be opened and the contents mixed with applesauce; capsules should not be crushed. A 48 year-old patient with suspected gastrointestinal (GI) bleeding undergoes diagnostic endoscopy, which determines the site of bleeding is a duodenal ulcer. The nurse explains to the patient that bleeding ulcers are most commonly related to which of the following? a. Intake of spicy foods b. Use of salicylates c. History of smoking d. Severe retching and vomiting - CORRECT ANSWER b. Use of salicylates Most bleeding ulcers are related to the presence of Helicobacter pylori or drug use, especially aspirin and aspirin-containing produces (salicylates) or nonsteroidal anti-inflammatory drugs. Severe retching and vomiting are associated with esophageal bleeding. Spicy floods do not cause ulcers. The nurse recognizes which of the following as appropriate interventions for the patient with acute pancreatitis? a. Manage pain and increase pancreatic secretions. b. Hydrate aggressively to decrease serum calcium levels. c. Administer corticosteroids to decrease pancreatic inflammation. d. Increase fluid volume and decrease pancreatic enzymes. - CORRECT ANSWER d. Increase fluid volume and decrease pancreatic enzymes. Maintaining circulation by increasing fluid volume and keeping the patient NPO to decrease the production of pancreatic enzymes is the appropriate action. Though pain management is a vital intervention for patients with pancreatitis, increased pancreatic secretions would not be a goal of care. Serum calcium may already be decreased in pancreatitis so the nurse would not manage fluid hydration in order to decrease value. Patients with pancreatitis often have glucosuria and hyperglycemia; corticosteroids will increase the likelihood that these conditions will develop and are not recommended in the treatment of acute pancreatitis. A patient is admitted to the medical-surgical unit with abdominal pain radiating to the right shoulder. The patient also complains of nausea and vomiting. A tentative diagnosis of cholecystitis with cholelithiasis has been made and the nurse prepares the patient for hepatobiliary imminodiacetic acid (HIDA) scanning. The nurse should instruct the patient to: a. Take the radioactive tracer by mouth prior to the procedure. b. Fast for 4 hours prior to the procedure. c. Limit fluids following the study. d. Take opioid analgesics immediately before the procedure. - CORRECT ANSWER b. Fast for 4 hours prior to the procedure. The patient should fast for at least 4 hours prior to the HIDA scan. Fluids should not be limited; some providers encourage the patient to drink water throughout the day to help flush the radioactive tracer from the body in the urine. Prescribed analgesics should not be taken 12 hours prior to the procedure. A patient with hepatitis C virus (HVC) asks the nurse how he could have contracted the disease. The nurse's response is based on awareness of the patient's risk factors, including which of the following: a. Use of injectable recreational drugs 10 years ago b. Long-term treatment with peritoneal dialysis c. Organ transplantation 5 years ago d. History of systemic lupus erythematosus (SLE) - CORRECT ANSWER a. Use of injectable recreational drugs 10 years ago Former drug users, including those who injected only once many years ago, are at risk for development of hepatitis C. Persons who underwent organ transplantation before 1992 may be at risk for HCV; however, better testing of donors became available in 1992. Peritoneal dialysis and SLE are not risk factors for HCV. In the patient with liver disease, the nurse recognizes which of the following as suggestive of portal hypertension: a. Abdominal striae and malodorous urine b. Increased white blood cell count c. Ascites and tarry stools d. Increased platelet count - CORRECT ANSWER c. Ascites and tarry stools Ascites and GI bleeding are symptoms of portal hypertension. Other symptoms include decreased platelets and decreased white blood cell count. Abdominal striae are often associated with the enlarged abdomen of pregnancy, or with Cushing's syndrome, diabetes, and long-time use of corticosteroid compounds. The nurse is caring for a patient with chronic liver disease. The nurse recognizes which of the following as a sign of worsening hepatic encephalopathy in this patient: a. Rapid, jerky movements b. Slurred speech c. Rancid breath d. Insomnia - CORRECT ANSWER b. Slurred speech Slurred speech, slowed or sluggish movement, and drowsiness are signs of worsening hepatic encephalopathy. An early sign often noticed by family members is sweet or musty breath. The other options are not signs of worsening encephalopathy. In discussing Crohn's disease with a newly diagnosed patient, the nurse should tell the patient that: a. A high-fat diet often is linked to its onset. b. Pain is commonly on the left side. c. Intestinal strictures commonly result. d. Oral gold compounds are prescribed for bowel inflammation. - CORRECT ANSWER c. Intestinal strictures commonly result. Crohn's disease carries a greater risk for stricture than ulcerative colitis due to intestinal wall thickening and formation of scar tissue. Diet is not linked to disease development. The pain of Crohn's disease is most often in the right lower quadrant, mimicking appendicitis, because that is the location of the terminal ileus. While immunosuppressants such as sulfasalazine (Azulfidine) or methotrexate may be used to decrease bowel inflammation, oral gold is not used to treat Crohn's disease. A patient with irritable bowel syndrome describes frequent episodes of incontinence related to diarrhea. In assessing the patient's potential dietary triggers, the nurse will ask about frequent consumption of which of the following foods? a. Lactose-reduced milk b. Unpeeled fruit. c. Cooked oatmeal d. Plain pasta - CORRECT ANSWER b. Unpeeled fruit. Fruit with peels contain insoluble fiber; they pass directly into the intestines, causing gas and diarrhea. Lactose can be a bowel irritant, so a lactose-reduced product should carry less risk of diarrhea. Oats and pasta are sources of soluble fiber that can prevent bowel spasms and episodes of diarrhea. Theodore Jackson, age 32, is admitted to the surgical unit following laparoscopic gastroplasty for weight loss. Mr. Jackson's preoperative weight was 643 pounds. In response to Mr. Jackson's question about changes in his stomach following this procedure, the nurse tells him the stomach is now the size of: a. A football b. An egg c. A softball d. An orange - CORRECT ANSWER b. An egg Following gastroplasty, the stomach is approximately the size of an egg. This is important for the patient to understand, as his food intake will be greatly reduced. Before Mr. Jackson is allowed to have oral fluids, the nurse knows the surgeon may confirm the integrity of the staple line by ordering: a. An esophagogastroduodenoscopy (EGD). b. An abdominal CT scan. c. A barium swallow study. d. A gastrografin study. - CORRECT ANSWER d. A gastrografin study. To verify that no leakage occurs into the peritoneal cavity from the staple line, the surgeon may order a gastrografin study. The other studies are not indicated for this purpose. The nurse recognizes which of the following as a sign of dumping syndrome following gastroplasty? a. Nausea b. Constipating c. Bradycardia d. Ileus - CORRECT ANSWER a. Nausea Nausea and vomiting, abdominal bloating, diarrhea, and vasomotor symptoms (flushing, increased heart rate, diaphoresis, lightheadedness) can be indicative of dumping syndrome. Because the altered gastrointestinal tract can place Mr. Jackson at risk for malnutrition after hospital discharge, the nurse should recommend: a. Rapid progression to a regular diet. b. Frequent high-calorie snacks. c. Daily chewable multivitamins. d. Broccoli and other nutrient-dense vegetables. - CORRECT ANSWER c. Daily chewable multivitamins. A daily multivitamin in chewable or liquid form is recommended. The patient will remain on a pureed diet for approximately 6 weeks. The patient should avoid high-calorie snacks and hard vegetables such as broccoli and cauliflower. A patient on the medical unit has been diagnosed with Clostridium difficile colitis. Which of the following precautions related to this diagnosis will be nurse include in the patient's plan of care?: a. Airborne b. Droplet c. Standard d. Contact - CORRECT ANSWER d. Contact Contact precautions are indicated for a patient with C. difficile infection. Standard precautions are implemented for all patients and are not specific to this diagnosis. The nurse is talking with a new graduate about the difference between gastric and duodenal ulcers. Which of the following would the nurse identify as characteristic of gastric ulcers? a. Peak occurrence is in persons ages 50-60. b. They often are associated with pancreatic disease. c. Pain typically occurs 2-4 hours after meals. d. Occurrence is greater in men. - CORRECT ANSWER a. Peak occurrence is in persons ages 50-60. The peak occurrence for gastric ulcers is in persons ages 50-60; the peak for peptic ulcer disease is ages 35-45. Gastric ulcers are associated with incompetent pyloric sphincter and bile reflux; peptic ulcers are associated with other diseases, including COPD, pancreatic disease, and hyperparathyroidism. The pain of gastric ulcers typically occurs 1-2 hours after meals. The occurrence of duodenal ulcers is greater in men; more women have gastric ulcers. The nurse enters the room as the client is beginning to have a tonic-clonic seizure. What action should the nurse implement first? A. Note the first thing the client does in the seizure B. Assess the size of the client's pupils C. Determine if the client is incontinent of urine or stool D. Provide the client with privacy during the seizure - CORRECT ANSWER A. Note the first thing the client does in the seizure The patient with migraine headaches has a seizure. After the seizure, which action can you delegate to the nursing assistant? A. Document the seizure. B. Perform neurologic checks. C. Take the patient's vital signs. D. Restrain the patient for protection. - CORRECT ANSWER C. Take the patient's vital signs. You are preparing to admit a patient with a seizure disorder. Which of the following actions can you delegate to the LPN/LVN? A. Complete admission assessment. B. Set up oxygen and suction equipment. C. Place a padded tongue blade at bedside. D. Pad the side rails before patient arrives. - CORRECT ANSWER B. Set up oxygen and suction equipment. The client is being admitted to rule out a brain tumor. Which classic triad of symptoms supports a diagnosis of brain tumor? A. Nervousness, metastasis to the lungs, and seizures B. Headache, vomiting, and papilledema C. Hypotension, tachycardia, and tachypnea D. Abrupt loss of motor function, diarrhea, and changes in taste - CORRECT ANSWER B. Headache, vomiting, and papilledema The client diagnosed with a brain tumor was admitted to the intensive care unit with decorticate posturing. Which indicates that the client's condition is becoming worse? A. The client has purposeful movement with painful stimuli B. The client has assumed adduction of the upper extremities C. The client is aimlessly thrashing in the bed D. The client has become flaccid and does not respond to stimuli - CORRECT ANSWER D. The client has become flaccid and does not respond to stimuli The nurse is assessing the client diagnosed with meningococcal meningitis. Which assessment data would warrant notifying the healthcare provider? A. Purpuric lesions on the face B. Complaints of light hurting the eyes C. Dull, aching, frontal headache D. Not remembering the day of the week - CORRECT ANSWER A. Purpuric lesions on the face You have just admitted a patient with bacterial meningitis to the medical-surgical unit. The patient complains of a severe headache with photophobia and has a temperature of 102.6°F orally. Which collaborative intervention must be accomplished first? A. Administer codeine 15 mg orally for the patient's headache. B. Infuse ceftriaxone (Rocephin) 2000 mg IV to treat the infection. C. Give acetaminophen (Tylenol) 650 mg orally to reduce the fever. D. Give furosemide (Lasix) 40 mg IV to decrease intracranial pressure. - CORRECT ANSWER B. Infuse ceftriaxone (Rocephin) 2000 mg IV to treat the infection. You are providing care for a patient with an acute hemorrhagic stroke. The patient's husband has been reading a lot about strokes and asks why his wife did not receive alteplase (T-pa). What is your best response? A. "Your wife was not admitted within the time frame that alteplase is usually given." B. "This drug is used primarily for patients who experience an acute heart attack." C. "Alteplase dissolves clots and may cause more bleeding into your wife's brain." D. "Your wife had gallbladder surgery just 6 months ago and this prevents the use of alteplase." - CORRECT ANSWER C. "Alteplase dissolves clots and may cause more bleeding into your wife's brain." The client diagnosed with atrial fibrillation has experienced a transient ischemic attack (TIA). Which medication would the nurse anticipate being ordered for the client on discharge? A. An oral anticoagulant medication B. A beta-blocker medication C. An anti-hyperuricemic medication D. A thrombolytic medication - CORRECT ANSWER A. An oral anticoagulant medication Which patient should you, as charge nurse, assign to a new graduate RN who is orienting to the neurologic unit? A. A 28 year-old newly admitted patient with spinal cord injury. B. A 67 year-old patient with stroke 3 days ago and left-sided weakness. C. An 85 year-old dementia patient to be transferred to long-term care today. D. A 54 year-old patient with Parkinson's who needs assistance with bathing. - CORRECT ANSWER B. A 67 year-old patient with stroke 3 days ago and left-sided weakness. Which of the following types of drugs might be given to control increased intracranial pressure (ICP)? A. Barbiturates B. Carbonic anhydrase inhibitors C. Anticholinergics D. Histamine receptor blockers - CORRECT ANSWER A. Barbiturates The nurse is teaching family members of a patient with a concussion about the early signs of increased intracranial pressure (ICP). Which of the following would she cite as an early sign of increased ICP? A. Decreased systolic pressure B. Headache and vomiting C. Inability to wake the patient with noxious stimuli D. Dilated pupils that don't react to light - CORRECT ANSWER B. Headache and vomiting The patient with multiple sclerosis tells the nursing assistant that after physical therapy she is too tired to take a bath. What is your priority nursing diagnosis at this time? A. Fatigue related to disease state. B. Activity Intolerance due to generalized weakness. C. Impaired Physical Mobility related to neuromuscular impairment. D. Self-care Deficit related to fatigue and neuromuscular weakness. - CORRECT ANSWER D. Self-care Deficit related to fatigue and neuromuscular weakness. The client diagnosed with Parkinson's disease is being discharged on Sinemet, carbidopa/levodopa, an antiparkinsonian drug. Which statement is the scientific rational for combining these medications? A. There will be fewer side effects with this combination than with carbidopa alone B. Dopamine D requires the presence of both of these medications to work C. Carbidopa crosses the blood brain barriers to treat Parkinson's Disease D. Carbidopa makes more levodopa available to the brain - CORRECT ANSWER D. Carbidopa makes more levodopa available to the brain A patient with Parkinson's disease has a nursing diagnosis of Impaired Physical Mobility related to neuromuscular impairment. You observe a nursing assistant performing all of these actions. For which action must you intervene? A. The NA assists the patient to ambulate to the bathroom and back to bed. B. The NA reminds the patient not to look at his feet when he is walking. C. The NA performs the patient's complete bath and oral care. D. The NA sets up the patient's tray and encourages patient to feed himself. - CORRECT ANSWER C. The NA performs the patient's complete bath and oral care. Which diagnostic test is used to confirm the diagnosis of Amyotrophic Lateral Sclerosis (ALS or Lou Gehrig's disease)? A. Electromyelogram (EMG) B. Muscle biopsy C. Serum creatinine kinase enzyme (CK) D. Pulmonary function test - CORRECT ANSWER B. Muscle biopsy A patient with a spinal cord injury (SCI) complains about a severe throbbing headache that suddenly started a short time ago. Assessment of the patient reveals increased blood pressure (168/94) and decreased heart rate (48/minute), diaphoresis, and flushing of the face and neck. What action should you take first? A. Administer the ordered acetaminophen (Tylenol). B. Check the Foley tubing for kinks or obstruction. C. Adjust the temperature in the patient's room. D. Notify the physician about the change in status. - CORRECT ANSWER B. Check the Foley tubing for kinks or obstruction. The client with a C-6 spinal cord injury (SCI) is admitted to the emergency department complaining of a severe pounding headache and has a BP of 180/110. Which intervention should the emergency department nurse implement? A. Keep the client flat in bed B. Dim the lights in the room C. Assess for bladder distention D. Administer a narcotic analgesic - CORRECT ANSWER C. Assess for bladder distention A patient who has been admitted to the medical unit with new-onset angina also has a diagnosis of Alzheimer's disease. Her husband tells you that he rarely gets a good night's sleep because he needs to be sure she does not wander during the night. He insists on checking each of the medications you give her to be sure they are the same as the ones she takes at home. Based on this information, which nursing diagnosis is most appropriate for this patient? A. Decreased Cardiac Output related to poor myocardial contractility. B. Caregiver Role Strain related to continuous need for providing care. C. Ineffective Therapeutic Regimen Management related to poor patient memory. D. Risk for Falls related to patient wandering behavior during the night. - CORRECT ANSWER B. Caregiver Role Strain related to continuous need for providing care. The nursing assistant reports to you, the RN, that the patient with myasthenia gravis (MG) has an elevated temperature (102.2°F), heart rate of 120/minute, rise in blood pressure (158/94), and was incontinent of urine and stool. What is your best first action at this time? A. Administer an acetaminophen suppository. B. Notify physician immediately. C. Recheck vital signs in 1 hour. D. Reschedule patient's physical therapy. - CORRECT ANSWER B. Notify physician immediately. The nurse must obtain surgical consent forms for the following clients who are scheduled for surgery. Which client would not be able to consent to surgery? A. The 65 year-old client who cannot read or write. B. The 30 year-old who does not understand English. C. The 16 year-old client who has a fractured ankle. D. The 80 year-old client who is not oriented to the day. - CORRECT ANSWER C. The 16 year-old client who has a fractured ankle. When preparing a client for surgery, which intervention should the nurse implement first? A. Check the permit for the spouse's signature. B. Take and document intake and output. C. Administer the "on call" sedative. D. Complete the preoperative checklist. - CORRECT ANSWER D. Complete the preoperative checklist. You are reviewing the complete blood count (CBC) for a client who has been admitted for knee arthroscopy. Which value is most important to report to the physician prior to surgery? A. White blood cell count 16,000/mm3 B. Hematocrit 33% C. Platelet count 426,000/mm3 D. Hemoglobin 10.9 g/dL - CORRECT ANSWER A. White blood cell count 16,000/mm3 Which nursing task can the nurse delegate to the unlicensed nursing assistant (NA)? A. Complete the preoperative checklist. B. Assess the client's preoperative vital signs. C. Teach the client about coughing and deep breathing. D. Assist the client to remove clothing and jewelry. - CORRECT ANSWER D. Assist the client to remove clothing and jewelry. The nurse is caring for a client scheduled for abdominal surgery. Which interventions should the nurse include in the plan of care? Select all that apply. A. Perform range-of-motion exercises. B. Discuss how to cough effectively. C. Explain how to perform deep-breathing exercises. D. Teach ways to manage postoperative pain. E. Discuss events that occur in the post-anesthesia care unit. - CORRECT ANSWER A, B, C, D, & E While completing the preoperative assessment, the male client tells the nurse that he is allergic to codeine. Which intervention should the nurse implement first? A. Apply an allergy bracelet on the client's wrist B. Label the client's allergies on the front of the chart C. Ask the client what happens when he takes the drug D. Document the allergy on the medication administration record - CORRECT ANSWER C. Ask the client what happens when he takes the drug The nurse is administering preoperative medication to a patient going to the operating room for an aortobifemoral bypass. After administering preoperative medication to the patient, the nurse should: A. Allow him to walk to the bathroom unassisted. B. Place the bed in low position with the side rails up. C. Tell him that he'll be asleep before he leaves for surgery. D. Take his vital signs. - CORRECT ANSWER B. Place the bed in low position with the side rails up. Which statement would be an expected outcome when the circulating nurse evaluates the goal of the intraoperative client? A. The client has no injuries from the OR equipment. B. The client has no postoperative infection. C. The client has stable vital signs during surgery. D. The client recovers from anesthesia. - CORRECT ANSWER A. The client has no injuries from the OR equipment. The client in the surgery holding area identifies the left arm as the correct surgical site, but the operative permits designate surgery to be performed on the right arm. Which interventions should the nurse implement? Select all that apply. A. Review the client's chart. B. Notify the surgeon. C. Immediately call a "time out." D. Change the surgical permit. E. Have the client mark the left arm. - CORRECT ANSWER A. Review the client's chart. B. Notify the surgeon. C. Immediately call a "time out." E. Have the client mark the left arm. A patient returns from the operating room after a small-bowel resection. Her vital signs are stable and her urine output averages 30 ml/hour. The nurse is reviewing the patient's fluid intake and output record. What is the normal relationship between fluid intake and urine output? A. Fluid intake should be double the urine output B. Fluid intake should approximately equal the urine output C. Fluid intake should be half the urine output D. Fluid intake should be inversely proportional to the urine output - CORRECT ANSWER B. Fluid intake should approximately equal the urine output A patient undergoes a surgical procedure that requires the use of general anesthesia. Following general anesthesia, the patient is most at risk for: A. Atelectasis B. Anemia C. Dehydration D. Peripheral edema - CORRECT ANSWER A. Atelectasis The nurse receives a report that the postoperative client received Narcan, an opioid antagonist, in PACU. Which client problem should the nurse add to the plan of care? A. Alteration in comfort. B. Risk for depressed respiratory pattern. C. Potential for infection. D. Fluid and electrolyte imbalance. - CORRECT ANSWER B. Risk for depressed respiratory pattern. Which data indicate the nursing care has been effective for the client who is one (1) day postoperative surgery? A. Urine output was 160 mL in the past eight (8) hours. B. Bowel sounds occur four (4) times per minute. C. T 99.0°F, P 98, R 20, and BP 100/60. D. Lungs are clear bilaterally in all lobes. - CORRECT ANSWER D. Lungs are clear bilaterally in all lobes. The charge nurse is making the shift assignments. Which postoperative client would be the most appropriate assignment to the graduate nurse? A. The 4 year-old client who had a tonsillectomy and is swallowing frequently B. The 74 year-old client with a repair of the left hip who is unable to ambulate C. A 24 year-old client who had an uncomplicated appendectomy the previous day D. An 80 year-old client with small bowel obstruction and congestive heart failure - CORRECT ANSWER C. A 24 year-old client who had an uncomplicated appendectomy the previous day Which statement would be an expected outcome for the postoperative client who had general anesthesia? A. The client will be able to sit in the chair for 30 minutes. B. The client will have a pulse oximetry reading of 97% on room air. C. The client will have urine output of 30 mL per hour. D. The client will be able to distinguish sharp from dull sensations. - CORRECT ANSWER B. The client will have a pulse oximetry reading of 97% on room air. The unlicensed nursing assistant reports the vital signs for a first-day postoperative client of T 100.8°F, P 80, R 24, and BP 148/80. Which intervention would be most appropriate for the nurse to implement? A. Administer the antibiotic earlier than scheduled. B. Change the dressing over the wound. C. Help the client turn, cough, and deep breathe every two (2) hours. D. Encourage the client to ambulate in the hall. - CORRECT ANSWER C. Help the client turn, cough, and deep breathe every two (2) hours. The nurse is caring for a patient who was given pain medication before leaving the recovery room. Upon returning to her room, the patient states that she's experiencing pain and requests more pain medication. Which is the best action for the nurse to take? A. Tell the patient that she must wait 4 hours for more pain medication. B. Give one-half of the ordered as-needed dose. C. Document the patient's pain. D. Notify the practitioner that the patient is continuing to experience pain. - CORRECT ANSWER D. Notify the practitioner that the patient is continuing to experience pain. The nurse is evaluating a patient postoperatively for infection. Which of the following would be most indicative of infection? A. The presence of an indwelling urinary catheter. B. A rectal temperature of 100°F (37.8°C). C. Redness, warmth, and tenderness in the incision area. D. A white blood cell (WBC) count of 8,000/ul - CORRECT ANSWER C. Redness, warmth, and tenderness in the incision area. The nurse is caring for a patient with a postoperative wound evisceration. Which action should the nurse perform first? A. Explain to the patient what is happening, and provide support. B. Cover the protruding organs with sterile gauze moistened with sterile saline solution. C. Push the protruding organs back into the abdominal cavity. D. Ask the patient to drink as much fluid as possible. - CORRECT ANSWER B. Cover the protruding organs with sterile gauze moistened with sterile saline solution. A patient, age 49, returns from the postanesthesia care unit after a total abdominal hysterectomy and bilateral salpingo-oophorectomy to treat cervical cancer. Which nursing intervention has the highest priority at this time? A. Monitor the patient for indications of hemorrhage. B. Assess the patient's pain level and response to analgesics. C. Encourage the patient to do deep breathing and leg exercises. D. Provide emotional support to the patient. - CORRECT ANSWER A. Monitor the patient for indications of hemorrhage. When preparing the plan of care for the client in acute pain as a result of surgery, the nurse should include which intervention? A. Administer pain medication as soon as the time frame allows B. Use nonpharmacological methods to replace medications C. Use cryotherapy after heat therapy because it works faster D. Instruct family members to administer medication with the PCA - CORRECT ANSWER A. Administer pain medication as soon as the time frame allows When administering an opioid narcotic, which interventions should the nurse implement to provide for client safety? Select all that apply. A. Compare the hospital number on the MAR to the client's bracelet. B. Have a witness verify the wasted portion of the narcotic. C. Assess the client's vital signs prior to administration. D. Determine if the client has any allergies to medications. E. Clarify all orders with the health-care provider. - CORRECT ANSWER A. Compare the hospital number on the MAR to the client's bracelet. C. Assess the client's vital signs prior to administration. D. Determine if the client has any allergies to medications. A new RN is preparing to administer packed red blood cells (PRBCs) to a client whose anemia was caused by blood loss after surgery. Which action by the new RN requires that you, as charge RN, intervene immediately? A. The new RN waits 20 minutes after obtaining the PRBCs before starting the infusion. B. The new RN starts an intravenous line for the transfusion using a 22-gauge catheter. C. The new RN primes the transfusion set using 5% dextrose in lactated Ringer's solution. D. The new RN tells the client that the PRBCs may cause a serious transfusion reaction. - CORRECT ANSWER C. The new RN primes the transfusion set using 5% dextrose in lactated Ringer's solution. What is gas exchange between the lungs and blood and between the blood and tissues? A. Diffusion B. Respiration C. Perfusion D. Ventilation - CORRECT ANSWER B. Respiration What should the nurse palpate when assessing for an upper respiratory tract infection? A. Neck lymph nodes B. Nasal mucosa C. Tracheal mucosa D. All of the above - CORRECT ANSWER A. Neck lymph nodes A classic finding in the patient with acute respiratory distress syndrome (ARDS) is: A. Sudden hypocalcemia with tetany B. Severe hyperkalemia C. Hypoxia resistant to oxygen therapy D. Hypercapnia - CORRECT ANSWER C. Hypoxia resistant to oxygen therapy Which medication should the nurse anticipate the health-care provider ordering for the client diagnosed with acute respiratory distress (ARDS)? A. An aminoglycoside antibiotic B. A synthetic surfactant C. A calcium channel blocker D. A nonsteroidal anti-inflammatory drug - CORRECT ANSWER B. A synthetic surfactant You are supervising a nursing student who is providing care for a thoracotomy client with a chest tube. What finding will you clearly instruct the nursing student to notify you about immediately? A. Chest tube drainage of 10-15 mL per hour B. Continuous bubbling in the water seal chamber C. Complaints of pain at the chest tube site D. Chest tube dressing dated yesterday - CORRECT ANSWER B. Continuous bubbling in the water seal chamber The patient diagnosed with an exacerbation of COPD is in respiratory distress. Which intervention should the nurse the nurse implement first? A. Assist the client into a sitting position at 90 degrees B. Give oxygen at 6 LPM via nasal cannula C. Monitor vital signs with the client sitting upright D. Notify the health-care provider about the patient's status - CORRECT ANSWER A. Assist the client into a sitting position at 90 degrees The client with COPD tells the nursing assistant that she did not get her annual flu shot this year and has not had a pneumonia vaccination. You will be sure to instruct the nursing assistant to report which of the following? A. Blood pressure 152/84 B. Respiratory rate 27/minute C. Heart rate 92/minute D. Oral temperature 101.2°F - CORRECT ANSWER D. Oral temperature 101.2°F A 24 year-old factory worker with a history of asthma presents to the emergency department with difficulty breathing. During the respiratory assessment of the patient, the nurse would expect to auscultate: A. diminished breath sounds and crackles in lung bases B. moist crackles in lung bases C. distant breath sounds and expiratory wheezes D. bronchial breath sounds in lung bases - CORRECT ANSWER C. distant breath sounds and expiratory wheezes A severe inflammatory response triggered a patient's asthma attack. Which of the following drugs would the nurse expect to give to combat the inflammatory response? A. theophylline (Aminophylline) B. prednisone (Sterapred) C. metoprolol (Lopressor) D. albuterol (Proventil) - CORRECT ANSWER B. prednisone (Sterapred) The patient diagnosed with exercise-induced asthma (EIA) is being discharged. Which information should the nurse include in the discharge teaching? A. Take 2 puffs on the rescue inhaler and wait five minutes before exercise B. Warm up exercises will increase the potential for developing the asthma attacks C. Use of bronchodilator inhaler immediately prior to beginning to exercise D. Increase dietary intake of food high in monosodium glutamate (MSG) - CORRECT ANSWER C. Use of bronchodilator inhaler immediately prior to beginning to exercise The client diagnosed with restrictive airway disease, asthma, has been prescribed a glucocorticoid inhaled medication. Which information should the nurse teach regarding this medication? A. Do not abruptly stop taking this medication; it must be tapered off. B. Immediately rinse the mouth following administration of the drug. C. Hold the medication in the mouth for fifteen (15) seconds before swallowing. D. Take the medication immediately when an attack starts. - CORRECT ANSWER B. Immediately rinse the mouth following administration of the drug. A patient has chronic bronchitis. The nurse is teaching him breathing exercises. Which point should the nurse include in her teaching? A. Make inhalation longer than exhalation. B. Exhale through an open mouth. C. Use diaphragmatic breathing. D. Use chest breathing. - CORRECT ANSWER C. Use diaphragmatic breathing. You are evaluating and assessing a client diagnosed with chronic emphysema. The client is on oxygen at a flow rate of 5L/min by nasal cannula. Which finding concerns you immediately? A. The client has fine bibasilar crackles. B. The client's respiratory rate is 8 breaths/minute. C. The client sits up and leans over the nightstand. D. The client has a large barrel chest. - CORRECT ANSWER B. The client's respiratory rate is 8 breaths/minute. You are initiating a nursing care plan for a client with pneumonia. Which intervention for cough enhancement should you delegate to the inexperienced nursing assistant? A. Teach the client about the importance of adequate fluid intake and hydration. B. Assist client to sitting position with neck flexed, shoulder relaxed and knees flexed. C. Remind the client to use incentive spirometry every 1-2 hours while awake. D. Encourage client to take a deep breath, hold it for 2 seconds, then cough 2-3 times in succession. - CORRECT ANSWER C. Remind the client to use incentive spirometry every 1-2 hours while awake. The nurse is planning the care of a client diagnosed with pneumonia and writes a problem of "impaired gas exchange." Which would be an expected outcome for this problem? A. Performs chest physiotherapy three (3) times a day. B. Able to complete activities of daily living. C. Ambulates in the hall and back several times during each shift. D. Alert and oriented to person, place, time, and events. - CORRECT ANSWER D. Alert and oriented to person, place, time, and events. The client diagnosed with a pulmonary embolus is being discharged. Which intervention should the nurse discuss with the client? A. Increase fluid intake to two (2) to three (3) liters a day. B. Eat a low-cholesterol, low-fat diet. C. Avoid being around large crowds. D. Receive pneumonia and flu vaccines. - CORRECT ANSWER A. Increase fluid intake to two (2) to three (3) liters a day. The employee health nurse is administering tuberculin skin testing to employees who have possibly been exposed to a client with active tuberculosis. Which statement indicates the need for radiological evaluation instead of skin testing? A. The client's first skin test indicates a purple flat area at the site of injection. B. The client's second skin test indicates a red area measuring four (4) mm. C. The client's previous skin test was read as positive. D. The client has never shown a reaction to the tuberculin medication. - CORRECT ANSWER C. The client's previous skin test was read as positive. The nurse observes the unlicensed nursing assistant (NA) entering an airborne isolation room and leaving the door open. Which action would be the nurse's best response? A. Close the door and discuss the NA's action when the NA comes out the room B. Make the NA come back outside the room and then reenter closing the door C. Say nothing to the NA but report the incident to the nursing supervisor D. Enter the patient's room and discuss the matter with the NA immediately - CORRECT ANSWER A. Close the door and discuss the NA's action when the NA comes out the room You are preparing a client with TB for discharge. Which statement by the client indicates that additional teaching is required? A. "All of my family members need to go and see the doctor for tuberculosis testing." B. "I will continue to take my isoniazid (INH) until I am feeling completely well." C. "I will cover my mouth and nose when I sneeze or cough and put my used tissues in a plastic bag." D. "I will change my diet to include more foods rich in iron, protein, and vitamin C." - CORRECT ANSWER B. "I will continue to take my isoniazid (INH) until I am feeling completely well." Ms. March's new orders call for a rapid-acting insulin by subcutaneous injection. The nurse will administer which of the following medications? A. Lispro (Humalog) B. Glargine (Lantus) C. Regular (Novolin R) D. Lente (Humulin L) - CORRECT ANSWER A. Lispro (Humalog) Lispro is the only rapid-acting insulin among the options. Glargine is a long-acting insulin, Regular insulin is short-acting, and Lente is intermediate-acting. The nurse is aware the onset of a rapid-acting insulin is 15 minutes, but how long after administration does the peak action occur? A. 15-45 minutes B. 60-90 minutes C. 2-3 hours D. 4-6 hours - CORRECT ANSWER B. 60-90 minutes Rapid-acting insulin peaks in 30-90 minutes and lasts 3-6 hours. The nurse is teaching Ms. March about insulin self-injection. The nurse knows which of the following is the site of fastest absorption? A. Upper outer arm B. Buttock C. Mid-lateral thigh D. Abdomen - CORRECT ANSWER D. Abdomen The Abdomen is the preferred site for insulin administration, and the site of fastest absorption. Ms. March experiences itching and erythema around the insulin injection site. The nurse correctly tells the patient: A. "This is an allergic reaction. I will call the provider about discontinuing the insulin." B. "You need to clean the site more carefully before you inject the insulin." C. "This reaction is usually a short-term response. I will call your provider for an antihistamine order." D. "You may have a site infection. I will call the provider for a topical antibiotic order." - CORRECT ANSWER C. "This reaction is usually a short-term response. I will call your provider for an antihistamine order." Local reactions may be self-limiting within the first 1-3 months, or may improve with a low dose of antihistamine. A true allergy is rare, generally resulting from the use of animal insulin. To assess Ms. March's long-term maintenance of serum glucose within the target range, the nurse will monitor results of which ordered laboratory test? A. Hemoglobin A1C B. Glucose tolerance testing C. Pancreatic enzymes D. Serum insulin - CORRECT ANSWER A. Hemoglobin A1C Hemoglobin A1C is useful in determining glycemic values over time. The test shows the amount of glucose attached to hemoglobin molecules over the life span of the individual red blood cell. Other tests are not appropriate for this purpose. A diabetic patient presents with hot and dry skin, rapid and deep respirations, and a fruity odor to his breath. As charge nurse, you observe the new graduate RN accomplishing all these patient tasks. Which one requires that you intervene immediately? A. The RN checks the patient's fingerstick glucose. B. The RN encourages the patient to drink orange juice. C. The RN checks the patient's order for sliding scale insulin. D. The RN assesses the patient's vital signs every 15 minutes. - CORRECT ANSWER B. The RN encourages the patient to drink orange juice. The signs and symptoms the patient is exhibiting are consistent with Hyperglycemia. The RN should not give the patient additional glucose. Answers (A, C, D) are all appropriate interventions for this patient. The RN should also notify the physician at this time. Which statement made by the client would make the nurse suspect that the client is experiencing hyperthyroidism? A. "I just don't seem to have any appetite anymore." B. "I have a bowel movement about every 3 to 4 days." C. "My skin is really becoming dry and coarse." D. "I have noticed that all my collars are getting tighter." - CORRECT ANSWER D. "I have noticed that all my collars are getting tighter." The thyroid gland (in the neck) enlarges as a result of the increased production of thyroid hormone; an enlarged gland is called a goiter. Answers (A, B, C) are all symptoms of Hypothyroidism Test-Taking Hint: If the test taker does not know the answer, sometimes thinking about the location of the gland or organ that is causing the problem may help the test taker select or rule out specific options. The nurse is providing dietary education for a patient with hyperthyroidism. Which of the following instruction by the nurse is appropriate for the patient? A. "Choose high-fiber food sources to stimulate the GI tract." B. "Avoid caffeine-containing liquids to decrease sleep disturbances." C. "Limit carbohydrate intake to allow use of stored protein." D. "Plan three full meals a day and take mineral supplements." - CORRECT ANSWER B. "Avoid caffeine-containing liquids to decrease sleep disturbances." Avoid Caffeine-containing foods such as coffee, tea, and cola should be avoided to decrease restlessness and sleep disturbances. High-fiber foods should be avoided because they stimulate the already hyperactive GI tract. A high-calorie diet of six full meals a day with snacks high in protein, carbs, minerals and vitamins should be planned. The nurse would anticipate which of the following as a possible complication of thyroidectomy? A. Hypocalcaemia, hemorrhage B. Tachycardia, anuria C. Hypotension, pleural effusion D. Hyperkalemia, hypovolemia - CORRECT ANSWER A. Hypocalcaemia, hemorrhage Hypocalcaemia results from the decrease in parathyroid hormone (PTH) secretion as a result of possible manipulation or accidental removal of parathyroid glands. Hemorrhage is most likely to occur in the first 24 hours after surgery. Which signs and symptoms would make the nurse suspect that the patient is experiencing a thyroid storm? A. Obstipation and hypoactive bowel sounds B. Hyperpyrexia and extreme tachycardia C. Hypotension and bradycardia D. Decreased respirations and hypoxia - CORRECT ANSWER B. Hyperpyrexia and extreme tachycardia Hyperpyrexia (high fever) and heart rate above 130 beats/minute are signs of Thyroid storm, a severely exaggerated form of hyperthyroidism. Answers (A, C, D) are all symptoms of Myxedema Coma (extreme form of Hypothyroidism) Test-Taking Hint: If the test taker does not have the knowledge to answer the question, then the test taker should look at the options closely. Answers A, C, and D all have the signs/symptoms of "decrease" - hypoactive, hypotension, and hypoxia. The test taker should select the option that does not match. The client is diagnosed with hypothyroidism. Which signs and symptoms would the nurse expect the client to exhibit? A. Complaints of extreme fatigue and hair loss B. Exophthalmos and complaints of nervousness C. Complaints of profuse sweating and flushed skin D. Tetany and complaints of stiffness of the hands - CORRECT ANSWER A. Complaints of extreme fatigue and hair loss A decrease in the thyroid hormone causes decreased metabolism, which leads to fatigue and hair loss. Answers (B, C) are signs of Hyperthyroidism. Answer (D) is a sign of Parathyroidism. Test-Taking Hint: Often if the test taker does not know the specific signs/symptoms of the disease but knows the function of the system that is affected by the disease, some possible answers can be ruled out. Tetany and stiffness of the hands are related to calcium, the level of which is influenced by the parathyroid gland, not the thyroid gland; therefore, answer D can be ruled out. All of the other 3 answers relate to metabolism, which is regulated by the thyroid gland. The test taker must decide which option lists symptoms of decreased thyroid function. Which medication order would the nurse question in the client diagnosed with untreated hypothyroidism? A. Thyroid hormones B. Oxygen C. Sedatives D. Laxatives - CORRECT ANSWER C. Sedatives Untreated hypothyroidism is characterized by an increased susceptibility to the effects of most hypnotic and sedative agents: therefore, the nurse would question this medication. Answers (A, B, D) are all appropriate interventions to give to someone with Hypothyroidism. Test-Taking Hint: When a question asks which order the nurse would question, three of the answers would be orders that the nurse would expect to administer to the client. Sometimes saying, "The nurse would administer this medication," may help the test taker select the correct answer. The client diagnosed with hypothyroidism is prescribed the thyroid hormone levothyroxine (Synthroid). Which assessment data indicates the medication has been effective? A. The client has a three (3)-pound weight gain. B. The client has a decreased pulse rate. C. The client's temperature is WNL. D. The client denies any diaphoresis. - CORRECT ANSWER C. The client's temperature is WNL. The client with Hypothyroidism frequently has a subnormal temperature, so a temperature WNL indicates the medication has been effective. Answers (A, B) indicate that the patient is not taking enough medication as they're still exhibiting signs of Hypothyroidism. Answer (D) is a symptom of Hyperthyroidism, so you'd expect to see a decrease in sweating in patients on medical management for Hyperthyroidism. Test-Taking Hint: One way of determining the effectiveness of medication is to determine if the signs/symptoms of the disease are no longer noticeable. A patient is admitted to the hospital with myxedema coma. The most critical nursing intervention for the patient at this time is: A. Administering an oral dose of levothyroxine (Synthroid) B. Warming the patient with a warming blanket C. Measuring and recording intake and output accurately D. Maintaining a patent airway - CORRECT ANSWER D. Maintaining a patent airway Because respirations are depressed in myxedema coma, maintaining a patent airway is the most critical nursing intervention. Ventilator support is usually needed. For Answers (A, C) Thyroid replacement medication will be administered IV, and although I&Os are important, it is not the most critical at this time. For Answer (B) Although Hypothermia is often associated with "Mixadema Coma", a warming blanket should NOT be used because it may cause vasodilation and shock. Gradual warming with blankets would be more appropriate. You assess a patient with Cushing's disease. For which finding will you notify the physician immediately? A. Purple striae present on abdomen and thighs B. Weight gain of 1 pound since the previous day C. +1 dependent edema in ankles and calves D. Crackles bilaterally in lower lobes of lungs - CORRECT ANSWER D. Crackles bilaterally in lower lobes of lungs The presence of crackles in the patient's lungs indicate excess fluid volume due to excess water and sodium reabsorption, and may be a symptom of pulmonary edema, which must be treated rapidly. Answers (A, B, C) Striae, weight gain, and dependent edema are common findings in patients with Cushing's disease. These findings should be monitored, but are not urgent. A 30-year-old female has been diagnosed with Cushing's syndrome. The nurse knows the patient is most likely to exhibit which symptoms? A. Hypertension and hirsutism B. Hyperkalemia and obesity C. Hypotension and anemia D. Menstrual irregularities and hypoglycemia - CORRECT ANSWER A. Hypertension and hirsutism Hypertension may result from fluid retention due to mineralcorticoid excess, and they may also exhibit hirsutism (abnormal hair growth). For Answers (B, C, D) while they may also exhibit truncal or generalized obesity as well as menstrual irregularities, it is patient's with Addison's Disease that would exhibit Hyperkalemia, Hypoglycemia and Hypotension (it is the opposite in Cushing's Syndrome). The nurse is providing discharge instructions for a patient with Cushing's syndrome. The nurse understands instructions for home care must include: A. Stress reduction and management strategies. B. Use of calcium supplementation to avoid tetany. C. Need for adequate calories due to increased metabolism. D. Safe handling and administration of Propylthiouracil (PTU). - CORRECT ANSWER A. Stress reduction and management strategies. The patient with Cushing syndrome is unable to react physiologically to stressors, so discharge teaching should include ways to reduce or manage stress; for example, the patient should avoid exposure to extreme temperatures, infections, and emotional disturbance. Answer (B) is indicated for the patient with Hypoparathyroidism. Answer (C, D) are instructions for patient's with Hyperthyroidism. The nurse is performing discharge teaching for a client diagnosed with Cushing's disease. Which statement made by the client demonstrates an understanding of the instructions? A. "I will be sure to notify my health-care provider if I start to run a fever." B. "Before I stop taking the prednisone, I will be taught how to taper it off." C. "If I get weak and shaky, I need to eat some hard candy or drink some juice." D. "It is fine if I continue to participate in weekend games of tackle football." - CORRECT ANSWER A. "I will be sure to notify my health-care provider if I start to run a fever." Cushing's syndrome predisposes the client to develop infections as a result of the immunosuppressive nature of the disease. (B) The client has too much cortisol; this client would not be receiving prednisone, a steroid medication. This is indicated for Addison's disease. (C) These are symptoms of hypoglycemia, which would not be expected in this client because this client has high glucose levels (Hyperglycemic). (D) The client is predisposed to osteoporosis and fractures. Contact sports should be avoided. Test-Taking Hint: If the test taker is not aware of the disease problem, And if all else fails, you can still answer this question correctly by referring to common standard discharge instructions - namely, notify the health-care provider of a fever. A patient is hospitalized with adrenocortical insufficiency (Addison's disease). Which nursing activity should you delegate to the nursing assistant? A. Remind patient to change positions slowly. B. Check the patient for muscle weakness. C. Teach the patient how to collect 24-hour urine. D. Plan nursing interventions to promote fluid balance. - CORRECT ANSWER A. Remind patient to change positions slowly. Patients with hypofunction of the adrenal glands often have hypotension and should be instructed to change positions slowly. Once a patient has been instructed, it is appropriate for the nursing assistant to remind the patient of those instructions. Answers (B, C, D) all involve assessing, teaching, and planning nursing care require more education and should be done by a licensed RN. The nurse is caring for a patient who has received long-term corticosteroid treatment for Addison's disease. The nurse is aware of the patient's risk for injury related to development of what potential complication related to this treatment. A. Osteogenesis imperfecta B. Osteosarcoma C. Osteoporosis D. Osteomalacia - CORRECT ANSWER C. Osteoporosis Long-term corticosteroid use (greater than 3 months) can contribute to the development of osteroporosis and the risk for fragility fractures. Answers (A, B, D) are all diseases that result in weak or brittle bones, but they are not caused by steroid use. Osteogenesis imperfecta = is a genetic disorder characterized by bones that break easily Osteosarcoma = is the most common type of bone cancer. Osteomalacia = softening of the bones caused by a deficiency of Vit D or Calcium The patient with adrenal insufficiency is to be discharged on prednisone 10 mg orally each day. What will you be sure to teach the patient? A. Report excessive weight gain or swelling to the physician. B. Rapid changes of position may cause hypotension. C. A diet with foods high in potassium may be beneficial. D. Signs of hypoglycemia may occur while taking this drug. - CORRECT ANSWER A. Report excessive weight gain or swelling to the physician. Rapid weight gain and edema are signs of excessive drug therapy, and the dose of the drug needs to be adjusted. Answers (B, C, D) - Hypertension, hyperkalemia, and hyperglycemia are common in patients with Addison's disease Adrenal Hypofunction, - Prednisone does not cause Hypotension, Hypokalemia or Hypoglycemia, and these symptoms are to be expected in someone with Addison's disease. The patient in Addisonian Crisis is unable to respond to stress. Before discharge, the nurse should instruct the patient and his family that during stress it'll be necessary to: A. Administer cortisone I.M. B. Drink 8 oz. (237 mL) of fluids C. Perform capillary blood glucose monitoring four times daily D. Continue to take his usual dose of hydrocortisone - CORRECT ANSWER A. Administer cortisone I.M. A - Patients with Addison's disease and their family members should know how to administer I.M. hydrocortisone during periods of stress. Answers (B, D) While it's important to keep well hydrated during stress, and importance of medication adherance, but the critical component of discharge planning in this situation is to know how and when to administer hydrocortisone I.M. Answer (C) Capillary blood glucose monitoring isn't indicated in this situation because the patient doesn't have Diabetes, and cortisol replacement doesn't cause insulin resistance. The nurse is caring for a patient with Central Diabetes Insipidus. The primary care provider has ordered intravenous fluid replacement. The nurse recognizes which of the following as an appropriate solution for this purpose? A. Hypertonic saline B. Dextrose 5% in water C. 0.9% saline D. Dextrose 5% in lactated ringers - CORRECT ANSWER B. Dextrose 5% in water B - In Acute DI, Hypotonic Saline (0.45% NaCl) or Dextrose 5% in water is given and titrated to replace urine output. Answers (A, C, D) are not appropriate treatments for DI. You are preparing a 24 year-old patient with Central Diabetes Insipidus for discharge from the hospital. Which statement indicates that the patient needs additional teaching? A. "I will drink fluids equal to the amount of my urine output." B. "I will weigh myself every day using the same scale." C. "I will wear my medical alert bracelet at all times." D. "I will gradually wean myself off the vasopressin." - CORRECT ANSWER D. "I will gradually wean myself off the vasopressin." D - The patient with Central DI requires life-long vasopressin therapy (DDAVP). Answers (A, B, C) are all statements that are appropriate for the home care of this patient. The nurse is admitting a patient diagnosed with Syndrome of Inappropriate Antidiuretic Hormone (SIADH). Which labs should be reported to the provider? A. Serum sodium of 112 mEq/L and headache B. Serum potassium of 5.0 mEq/L and heightened awareness C. Serum calcium of 10 mg/dL and tented tissue turgor D. Serum magnesium of 1.2 mg/dL and large urine output - CORRECT ANSWER A. Serum sodium of 112 mEq/L and headache A - A serum sodium level of 112 mEq/L is dangerously low, and the client is at risk for seizures. A headache is a symptom of a low sodium level. NOTE: Another sign and symptom of hyponatremia is abdominal cramps. (B) This is a normal potassium level, and a heightened level of awareness could indicate drug usage. (C) This is a normal calcium level and the client is fluid overloaded, not dehydrated, so you would not see tented tissue turgor. (D) This is a normal magnesium level, and a large urinary output would be desired. Test-Taking Hint: The nurse must know common laboratory values. As part of the plan of care for a patient with SIADH, the nurse will: A. Initiate fluid restriction of 800-1000 mL/day. B. Maintain the head of the bed no more than 30 degrees. C. Administer intravenous hypotonic saline. D. Maintain the patient in a private room. - CORRECT ANSWER A. Initiate fluid restriction of 800-1000 mL/day. A - Fluid restriction of 800-1000 mL/day may be the only treatment needed for SIADH; this should result in gradual weight reduction, progressive rise in serum sodium concentration, and improvement in symptoms. Answer (B) - The head of the bed should be elevated no more than 10 degrees to enhance venous return to the heart and increase left atrial filling pressure, thus reducing the release of Anti-Diuretic Hormone. Answer (C) - Hypertonic saline may be slowly administered in the presence of neurologic symptoms associated with severe hyponatremia. Answer (D) - A private room is not indicated for a patient with SIADH. The nurse is teaching a patient diagnosed with basal cell carcinoma. The most common cause of basal cell carcinoma is: A. immunosuppression B. radiation exposure C. sun exposure D. burns - CORRECT ANSWER C. sun exposure A patient received burns to his entire back and left arm. Using the Rule of Nines, the nurse can calculate that he has sustained burns on what percentage of his body? A. 9% B. 18% C. 27% D. 36% - CORRECT ANSWER C. 27% A patient is admitted to a burn intensive care unit with extensive full thickness burns. The nurse is most concerned about the patient's: A. fluid status B. risk of infection C. body image D. level of pain - CORRECT ANSWER A. fluid status A patient undergoes a biopsy to confirm a diagnosis of skin cancer. Immediately following the procedure, the nurse should observe the site for: A. an infection B. dehiscence C. hemorrhage D. swelling - CORRECT ANSWER C. hemorrhage The nurse is caring for a patient with malignant melanoma. The nurse explains that the first and most important treatment for malignant melanoma is: A. chemotherapy B. immunotherapy C. wide excision D. radiation therapy - CORRECT ANSWER D. radiation therapy The client comes into the emergency room in severe pain and reports that a boiling pot of hot water accidentally spilled on his lower legs. The assessment reveals blistered, mottled red skin, and both feet are edematous. Which depth of burn should the nurse document? A. Superficial partial thickness B. Deep partial thickness C. Full thickness D. First degree - CORRECT ANSWER B. Deep partial thickness The client has full-thickness burns to 65% of the body, including the chest area. After establishing a patent airway, which collaborative intervention is priority for the client? A. Replace fluids and electrolytes B. Prevent contractures of extremities C. Monitor urine output hourly D. Prepare to assist with an escharotomy - CORRECT ANSWER A. Replace fluids and electrolytes The school nurse is preparing to teach a health promotion class to high school seniors. Which information regarding self-care should be included in the teaching? A. Wear a sunscreen with a protection factor of ten (10) or less when in the sun B. Try to stay out of the sun between 0300 and 0500 daily C. Perform a thorough skin check monthly D. Remember that caps and long sleeves do not help prevent skin cancer - CORRECT ANSWER C. Perform a thorough skin check monthly Which client is at the greatest risk for the development of skin cancer? A. The African American male who lives in the northeast B. The elderly Hispanic female who moved from Mexico as a child C. The client who has a family history of basal cell carcinoma D. The client with fair complexion who cannot get a tan - CORRECT ANSWER D. The client with fair complexion who cannot get a tan The client is admitted to the outpatient surgery center for removal of a malignant melanoma. Which assessment data indicate the lesion is a malignant melanoma? A. The lesion is asymmetrical and has irregular borders B. The lesion has a waxy appearance with pearl-like borders C. The lesion has a thickened and scaly appearance D. The lesion appeared as a thickened area after an injury - CORRECT ANSWER A. The lesion is asymmetrical and has irregular borders The client comes to the emergency department complaining of pain in the left lower leg following a puncture would from a nail in a board. The left lower leg is reddened with streaks, edematous and hot to the touch, and the client has a temperature of 100.8 degrees F. Which condition would the nurse suspect the client is experiencing? A. Cellulitis B. Lyme disease C. Impetigo D. Deep vein thrombosis - CORRECT ANSWER A. Cellulitis The school nurse is discussing impetigo with the teachers in an elementary school. One of the teachers asks the nurse, "How can I prevent getting impetigo?" Which statement would be the most appropriate response? A. "Wash your hands after using the bathroom." B. "Do not touch any affected areas without gloves." C. "Apply a topical antibiotic to your hands." D. "Keep the child with impetigo isolated in the room." - CORRECT ANSWER B. "Do not touch any affected areas without gloves." The nurse is discussing the prevention of herpes simplex 2. Which intervention should the nurse discuss with the client? A. Encourage the client to get the chickenpox vaccine. B. Do not engage in oral sex if you have a cold sore on the mouth C. Wear non-sterile gloves when cleaning the genital area. D. Do not share any type of towel or washcloth with another person. - CORRECT ANSWER B. Do not engage in oral sex if you have a cold sore on the mouth The nurse is assessing a young mother who came to the clinic complaining of sores on her skin. Which assessment data would support that the client has chickenpox? A. Crops of lesions that have pus and reddened base B. Oval scaling lesions that occur on the legs and arms C. Severe itching of the scalp with tiny eggs visible D. Ringed red lesions on the face, neck, trunk, and extremities - CORRECT ANSWER A. Crops of lesions that have pus and reddened base The school nurse is assessing a teacher who has pediculosis. Which statement by the teacher makes the nurse suspect that the teacher did not comply with the instructions that were discussed in the classroom with the children? A. I used the comb to remove all the nits B. I washed my hair with Kwell shampoo C. I removed all the sheets from my bed D. I had to fix my daughter's hair with my brush - CORRECT ANSWER D. I had to fix my daughter's hair with my brush As charge nurse, you are observing a newly hired RN. Which action by the new RN requires your most immediate action? A. Obtaining an anaerobic culture of a superficial partial-thickness arm burn B. Administration of tetracycline with a glass of milk to a client with cellulitis C. Debridement of a deep partial-thickness burn wound using wet-to-dry dressings D. Teaching a newly admitted burn client about the use of pressure garments - CORRECT ANSWER B. Administration of tetracycline with a glass of milk to a client with cellulitis The paraplegic client is being admitted to a medical unit from home with a stage IV pressure ulcer over the right ischium. Which assessment tool should be completed on admission to the hospital? A. Complete a Braden Scale B. Monitor the client on a Glasgow Coma Scale C. Assess for a Babinksi sign D. Initiate a Brudzinski flow sheet - CORRECT ANSWER A. Complete a Braden Scale The wound care nurse documented a client's pressure ulcers on admission as 3.3 X 4.0 cm Stage II on the coccyx. Which information would alert the nurse that the client's pressure ulcer is getting worse? A. Skin is not broken and is 2.5 X 3.4 cm with erythema that does not blanch B. There is a 3.2 X 4.1 cm blister that is red and drains occasionally C. The skin covering the coccyx is intact but the client complains of pain in the area D. The coccyx wound extends to the subcutaneous layer and there is drainage - CORRECT ANSWER D. The coccyx wound extends to the subcutaneous layer and there is drainage The nurse is caring for a client who has developed Stage IV pressure ulcers on the left trochanter and coccyx. Which collaborative problem had the highest priority? A. Impaired cognition B. Altered nutrition C. Self-care deficit D. Altered coping - CORRECT ANSWER B. Altered nutrition The nurse is caring for a patient with a sacral pressure ulcer with wet-to-dry dressing. Which guideline is appropriate when caring for a patient with wet-to-dry dressing? A. Wound should remain moist from dressing B. Dressing should be tightly packed C. Dressing should be allowed to dry before removal D. Dressing should be covered with plastic sheet dressing - CORRECT ANSWER C. Dressing should be allowed to dry before removal [Show More]

Last updated: 6 months ago

Preview 1 out of 102 pages

Reviews( 0 )

$10.00

Add to cart

Instant download

Can't find what you want? Try our AI powered Search

OR

GET ASSIGNMENT HELP
112
0

Document information


Connected school, study & course


About the document


Uploaded On

Oct 29, 2023

Number of pages

102

Written in

Seller


seller-icon
Nolan19

Member since 2 years

10 Documents Sold


Additional information

This document has been written for:

Uploaded

Oct 29, 2023

Downloads

 0

Views

 112

Recommended For You


$10.00
What is Browsegrades

In Browsegrades, a student can earn by offering help to other student. Students can help other students with materials by upploading their notes and earn money.

We are here to help

We're available through e-mail, Twitter, Facebook, and live chat.
 FAQ
 Questions? Leave a message!

Follow us on
 Twitter

Copyright © Browsegrades · High quality services·